Glaucoma (No=378)
Complications of ALT include all of the following EXCEPT:
synechial angle closure
cataract formation
post-operative IOP spike
iritis
Your answer was INCORRECT
Explanation
Complications of ALT include:
iritis
postoperative IOP spike
anterior synechiae formation
ALT is not associated with cataract formation.
All are true of pilocarpine EXCEPT:
it can be used to treat accommodative esotropia
causes brow ache
reverses phenylephrine mydriasis more effectively than thymoxamine
reverses atropine mydriasis
Your answer was CORRECT
Explanation
Pilocarpine causes contraction of the ciliary muscle thereby keeping the eye in an accommodative
state. As such, it can be used to treat accommodative esotropia by reducing the drive for
convergence. Pilocarpine can reverse atropine mydriasis. It can also reverse phenylephrine
mydriasis according to various sources but not as effectively as thymoxamine.
In a patient with acute, congestive, angle-closure glaucoma and a pressure of 70mmHg, which of
the following is LEAST likely to break the attack?
laser iridotomy
oral hyperosmotic agents
compression gonioscopy
topical miotic treatment
Your answer was INCORRECT
Explanation
In some mild cases, topical pilocarpine can pull the peripheral iris away from the trabecular
meshwork and break an attack of angle-closure. However, when the intraocular pressure is highly
elevated, as in this case, the iris is unlikely to respond to miotics due to iris ischaemia affecting the
sphincter muscle. In such cases, IOP should first be lowered by other medications (topical, oral or
iv), such as beta blockers, carbonic anhydrase inhibitors, and/or hyperosmotic agents.
Compression gonioscopy is a second- or third-line treatment in unresponsive cases and may help
to burp aqueous through the pupil, thereby breaking the pupillary block.
Laser iridotomy is the definitive treatment of choice for angle-closure, and is usually undertaken
once the IOP has been initially lowered. If laser cannot be accomplished because of corneal
clouding, surgical iridectomy or emergency lens extraction should be considered.
A patient presents 1 day post-trabeculectomy with a shallow anterior chamber. The bleb is well
formed, the sclerostomy and the peripheral iridotomy are patent. The intra-ocular pressure is
3mmHg.
Which of the following surgical techniques is LEAST likely to play a role in preventing a shallow
anterior chamber at day 1 after a trabeculectomy?
tight suturing of the conjunctival bleb at the limbus
mydriatics
paracentesis for maintaining intra-operative AC depth
thick scleral flap
tight suturing of the scleral flap
Your answer was INCORRECT
Explanation
Tight suturing of both the scleral and conjunctival flaps prevents early over-drainage and AC
shallowing. A paracentesis ensures a formed anterior chamber at the end of the procedure,
preventing early shallowing of the anterior chamber and aqueous misdirection. Ideal scleral flap
thickness is between one-third to one-half thickness. A thick flap provides more resistance to flow,
while a thin flap can cause excessive drainage and early hypotony.
Mydriatics rotate the ciliary body and can certainly play a role in relieving aqueous misdirection
were it to occur; however of all the available options above, they are least significant in preventing
early AC shallowing.
This question came in the FRCS (Glasgow) Part 2 in October 2014.
Primarily, miotics lower intraocular pressure by:
ciliary muscle contraction
increasing uveo-scleral outflow
reducing aqueous production by the ciliary body
moving peripheral iris away from the anterior chamber angle
Your answer was INCORRECT
Explanation
In plateau iris syndrome, miotics work by moving peripheral iris out of the anterior chamber
angle, thereby enhancing outflow facility. However, more generally, the primary mode of action
of miotics is through ciliary muscle contraction. When the ciliary muscle contracts it pulls on its
insertion at the scleral spur, in turn putting traction on the trabeculum, thereby increasing
conventional outflow.
In which condition would a laser peripheral iridectomy NOT be indicated?
prophylaxis in an eye with narrow angles
secclusio pupillae
neovascular glaucoma
acute angle-closure glaucoma
Your answer was CORRECT
Explanation
Neovascular glaucoma is caused by the growth of new vessels into the angle. This growth requires
panretinal photocoagulation to cause regression of the vessels, not a peripheral iridectomy.
Peripheral iridectomy in this context may simply precipitate further anterior chamber flare and
haemorrhage.
A 29-year-old myopic man presents with bilateral concave-shaped irides, heavily pigmented
trabecular meshworks and IOPs of 33 mmHg.
What is the most likely diagnosis?
plateau iris
pseudoexfoliation syndrome
primary angle closure
pigmentary glaucoma
Your answer was CORRECT
Explanation
The findings are most consistent with pigmentary glaucoma. It is more common in myopes, with
concave shaped irides that cause iris chaffing against the lens and zonules. Other findings are iris
transillumination and Krukenberg's spindle.
Iris transillumination defects are characteristic of all the following conditions EXCEPT:
plateau iris syndrome
pigment dispersion syndrome
oculocutaneous albinism
pseudoexfoliation syndrome
Your answer was CORRECT
Explanation
Plateau iris is not associated with transillumination defects.
Iris transillumination defects suggest:
oculocutaneous albinism (diffuse transillumination of the iris)
pseudoexfoliation (peripupillary transillumination defects)
pigment dispersion (peripheral, radial, discrete or confluent defects)
herpes zoster keratouveitis (sectoral atrophy)
herpes simplex keratouveitis (moth-eaten peri-pupillary defects)
iatrogenic (usually cataract surgery)
trauma
Rubeosis iridis usually develops initially:
at the pupillary margin
on the posterior iris surface
at the irido-corneal angle
in the mid-periphery
Your answer was INCORRECT
Explanation
Rubeosis iridis occurs initially at the pupil margin. It is detectable under high magnification as
dilated capillary tufts or red spots at the pupil margin. These join up and then grow radially over
the surface of the iris towards the angle. Angle neovascularization can, less commonly, occur in
the absence of iris neovascularization.
The association of neurofibroma with congenital glaucoma is strongest with lesions of the:
forehead
lower lid
nose
upper eyelid
Your answer was CORRECT
Explanation
Upper lid neurofibromas, especially plexiform neurofibromas (shown above), are most closely
related to the development of glaucoma in patients with NF-1.
In which condition is argon laser trabeculoplasty LEAST likely to be effective:
pigmentary glaucoma
Possner-Schlossman syndrome
pseudoexfoliation glaucoma
normal tension glaucoma
Your answer was CORRECT
Explanation
ALT is successful in:
primary open angle glaucoma
pigmentary glaucoma
pseudoexfoliation glaucoma
normal tension glaucoma
It is generally ineffective in:
paediatric glaucomas
secondary glaucomas
Stevens-Johnson syndrome is a recognised side-effect of which class of IOP-lowering agent:
carbonic-anhydrase inhibitors
beta-blockers
prostaglandin analogues
alpha-2 agonists
Your answer was CORRECT
Explanation
Carbonic anhydrase inhibitor (CAI) side-effects include:
Paraesthesia of fingers, toes, hands and feet
Malaise complex: fatigue, depression, weight loss, reduced libido
GI complex: diarrhea, nausea, cramps
Renal stone formation
Steven-Johnson syndrome
Blood dyscrasias: bone marrow suppression, aplastic anaemia.
All are true of pigment dispersion EXCEPT:
Sampaolesi's line occurs anterior to Schwalbe's line
patients may report blurred vision after excercise
the rate of pigment deposition increases with age
it is usually diagnosed in the third or fourth decades
Your answer was INCORRECT
Explanation
The rate of pigment deposition in pigment dispersion syndrome actually decreases in age, likely
due to the increasing size of the lens which lifts the iris away from the zonules. Other options
supplied above are true.
Which genus of organism is most commonly isolated in bleb-associated endophthalmitis:
Streptococcus
Neisseria
Propionibacterium
Enterococcus
Staphylococcus
Your answer was INCORRECT
Explanation
Acute bleb-associated endophthalmitis can occur at any time following successful filtration
surgery. Early blebitis (within 6 weeks of surgery) is usually caused by Staph. epidermidis. Late
blebitis (over 6 weeks) is usually caused by Pneumococcus (Streptococcus pneumoniae) and
Haemophilus influenzae. Overall, the most common cause of bleb-associated endophthalmitis are
Streptococcus species. Reference
All of the following are true of iridoschisis EXCEPT:
it occurs in the elderly
it is associated with a shallow anterior chamber
it is most prominent in the superior iris
gonioscopy may reveal peripheral anterior synechae
Your answer was INCORRECT
Explanation
Iridoschisis is thought to result from intermittent angle-closure, which causes iris atrophy and
schisis due to the high pressure. Findings include shallow anterior chamber with occludable angles
on gonioscopy. Iridoschisis is usually most prominent in the inferior iris.
A 60-year-old phakic hyperopic woman undergoes trabeculectomy for uncontrolled POAG. On
the first postoperative day, her IOP is 10 mmHg, a diffuse bleb is present, and the anterior
chamber is deep. However, on postoperative day 3 she is noted to have a completely flat anterior
chamber, no bleb, and an IOP of 45 mmHg.
At this point, appropriate medical management includes each of the following EXCEPT:
systemic and topical aqueous suppressants
miotic medications
topical cycloplegic medications
topical corticosteroids
Your answer was INCORRECT
Explanation
The most likely diagnosis in this clinical setting is aqueous misdirection. Patients at greatest risk
include those with shallow anterior chambers: older patients, women, and hyperopes. Maximum
cycloplegia and aqueous suppression may help to break the cycle of aqueous being misdirected
into the vitreous cavity. Half of eyes with aqueous misdirection may be successfully managed
medically. Pseudophakic eyes can, on occasion, be treated successfully with Nd:YAG laser to
disrupt the posterior capsule and/or anterior hyaloid face. The rest will require surgical
intervention by pars plana vitrectomy with or without lensectomy.
3-days after routine trabeculectomy a patient presents with an IOP of 1 mmHg and a flat anterior
chamber. The bleb is flat. The retina and choroid appear normal.
What is the MOST likely cause of these findings?
malignant glaucoma
choroidal detachments
excessive filtration
bleb leak
Your answer was CORRECT
Explanation
The most likely cause of the findings above is a bleb leak. Another possibility is ciliary body
shutdown post-surgery (not supplied as an option above) and a Seidel's test is useful to
discriminate between the two.
Of the options supplied, both suprachoroidal haemorrhage and malignant glaucoma will present
with a flat AC and a high (not low) pressure. Excessive filtration could have a low pressure and a
flat AC, but in this case the bleb would be exuberant (not flat).
Posner-Schlossman syndrome:
requires systemic or periocular corticosteroid therapy
is often self-limiting
is typically painless
is associated with acute rather than late-onset glaucoma
Your answer was INCORRECT
Explanation
Posner-Schlossman syndrome is characterised by:
unilateral mild ocular pain or photophobia (less commonly it can be painless)
mild anterior uveitis
elevated IOP
self-limited episodes
While self-limiting, the episodes often require topical glaucoma medications to control IOP. Mild
topical corticosteroids may be used to control intraocular inflammation. Systemic and periocular
corticosteroids are not indicated. Recurrent attacks of this syndrome may lead to eventual optic
nerve damage and late-onset glaucoma.
An abnormally prominent Schwalbe line visible without gonioscopy is referred to as:
posterior embryotoxon
Axenfeld's anomaly
Peters' anomaly
Rieger's anomaly
Your answer was INCORRECT
Explanation
Figure: Posterior embryotoxon
Posterior embryotoxon is an anteriorly displaced Schwalbe line, visible without gonioscopy.
Axenfeld's anomaly is posterior embryotoxon and attached iris processes.
Rieger's anomaly is the components of Axenfeld's anomaly plus atrophy of the iris stroma.
Rieger's syndrome is Rieger's anomaly with skeletal, dental, or craniofacial abnormalities.
Note, however, that despite the above definitions, nowadays the term Axenfeld-Rieger syndrome is
considered more appropriate given the high incidence of overlapping findings, rather than rigidly
defining each syndrome/anomaly.
All of the following are recognised complications of glaucoma tube shunt procedures EXCEPT:
hypotony
corneal neovascularization
conjunctival melt
diplopia
Your answer was INCORRECT
Explanation
Complications of glaucoma implant surgery include:
hypotony
shallow chamber
migration/expulsion of tube
conjunctival melts
corneal endothelial failure
diplopia
elevated IOP
Which one of the following statements about perimetry testing is TRUE?
static perimetry is the most useful for quantifying and tracking visual field changes in a
patient with established glaucoma
the Goldmann visual field is a form of static perimetry
early, specific signs of glaucoma include generalised constriction of isopters and baring of the
blind spot
in static perimetry, the stimulus intensity is held constant (static) and moved centrally until it is
detected
Your answer was CORRECT
Explanation
In static perimetry (such as the Humphrey), the stimulus is of variable intensity and is kept
stationary (static) until it is noticed by the patient. The Goldmann field is a form of kinetic
perimetry, with a moving target. Baring of the blind spot and generalised constriction are not very
specific and can be produced by miosis, uncorrected refractive error, aging, and cataract.
All of the following contact lenses for gonioscopy are examples of indirect goniolenses EXCEPT:
Sussman lens
Goldmann lens
Posner lens
Koeppe lens
Your answer was CORRECT
Explanation
With direct gonioscopy, the angle is visualized directly through the contact lens; while with
indirect gonioscopy, light rays are reflected by a mirror in the contact lens.
Direct lenses include the: Koeppe, Richardson, Barkan, Wurst and Swan-Jacob lenses.
Indirect lenses include the: Goldmann, Zeiss, Posner and Sussman lenses.
Direct lenses are often used for surgical procedures because there is direct visualisation of the
meshwork without the image inversion that accompanies indirect (mirrored) lenses.
Indirect lenses are the most commonly used in daily practice because of their ease of use on slit-
lamp machines. Among the indirect lenses, there are two main types: large scleral-type lenses (e.g.
Goldmann 3-mirror) and smaller, corneal-type lenses (e.g. Posner, Zeiss, Sussman).
Scleral-type lenses:
larger
require a coupling solution
excellent lens stability
good eyelid control
stable image
better view where corneal opacities present
3-mirror allows peripheral, midperipheral and posterior pole views if required
limitations for dynamic indentation gonioscopy
pressure on the lens will falsely narrow the angle by pushing on the sclera
Corneal-type lenses:
smaller
no coupling solutuion required
no lens rotation as all 4 mirrors look into the angles
better patient co-operation needed versus a scleral-lens
unintentional compression of the cornea creates striae and distorts view
unintentional compression falsely opens the angle by pushing aqueous into angle
dynamic indentation gonioscopy (by intentionally compressing the cornea) can differentiate
appositional vs synechial angle closure which is very useful clinically
Side-effects of the prostaglandin analogue drops include all of the following EXCEPT:
increased iris pigmentation
conjunctival melanosis
hypertrichosis
conjunctival hyperaemia
cystoid macular oedema
Your answer was CORRECT
Explanation
The most frequent side-effects of the prostaglandin analogues include:
increased iris pigmentation
iris cyst formation
eyelash hypertrichosis
conjunctival hyperaemia
increase in peri-ocular skin pigmentation
peri-orbital fat atrophy
iritis
CMO
Conjunctival melanosis was classically associated with topical epinephrine, but this drug is no
longer routinely used in glaucoma.
Which is TRUE regarding the use of the Nd:YAG laser by comparison to the argon laser for
performing iridotomies?
less frequent bleeding with application
fewer total applications
more extensive histologic damage around the treatment site
more late closure of the iridotomy
Your answer was INCORRECT
Explanation
Iridotomies created with an argon laser:
require more applications
greater energy
more extensive early oedema and tissue destruction at the margins
more iritis
more pupillary distortion
higher incidence of late closure of the iridotomy
Clinically, the Nd:YAG laser has the disadvantage of frequent bleeding.
In some cases, (e.g. very dark irides) it may be advantageous to use both lasers: the argon for its
coagulative effects and the Nd:YAG for its disruptive properties.
A 65-year old man presents with a 1-week history of severe right-sided headache with nausea and
blurred vision. On examination, he has ciliary injection, right corneal Descemet's folds with
moderate stromal oedema. He has right-sided keratitic precipitates, flare and glaukomflecken.
Intra-ocular pressures are 15mm Hg bilaterally.
What is the most important diagnostic test to confirm the diagnosis?
dilated fundoscopy
auto-antibody screen
fluorescein angiography
gonioscopy
10% phenylephrine
Your answer was CORRECT
Explanation
This gentleman's symptoms are consistent with post-congestion angle closure, and this is
confirmed by the presence of glaukomflecken, which is pathognomic for previous angle closure.
His IOP has spontaneously returned to normal - either due to spontaneous re-opening of the angle,
or due to ciliary body ischaemia. The most important diagnostic test in this instance is gonioscopy,
which should confirm bilaterally shallow angles, with possible right-sided iridocorneal touch.
A 34-year-old male complains of intermittent blurry vision in his left eye every time he plays
tennis. On examination, the IOP is normal, there is partial loss of the iris pupillary ruff and mid-
peripheral iris transillumination defects bilaterally.
What is the most likely the diagnosis?
plateau iris syndrome
pigment dispersion syndrome
iridocorneal endothelial syndrome
Posner-Schlossman syndrome
Your answer was CORRECT
Explanation
Figure: mid-peripheral iris transillumination
Mid-peripheral iris transillumination is suggestive of pigment dispersion syndrome. Exercise may
induce temporary blurring with or without a spike in PDS due to release of pigment into the
anterior chamber.
Which of the following is characteristic of pseudo-exfoliation?
autosomal recessive inheritance
more common in females
approximately 50% lifetime risk of glaucoma
secondary closed angle glaucoma
Your answer was CORRECT
Explanation
Pseudo-exfoliation has no clear inheritence pattern. It is more common in females and those of
Scandinavian ancestry. It causes a chronic open angle glaucoma in about 15% at 10 years.
This question came in the 2014 FRCOphth.
Which statement is FALSE regarding the treatment of primary open angle glaucoma:
trabeculectomy is more likely to be successful in black patients
the IOP-lowering effect of laser trabeculoplasty is usually transient, lasting a few years
topical IOP-lowering medication should be commenced as monotherapy
systemic side-effects of topical medication can be reduced by applying pressure to the medial
canthus after instillation of drops
Your answer was CORRECT
Explanation
Surgical trabeculectomy is less likely to succeed in black patients due to increased tendency
towards scarring and fibrosis. Other options are true.
Most cases of primary congenital glaucoma are:
autosomal dominant
sporadic
X-linked recessive
autosomal recessive
Your answer was CORRECT
Explanation
Most cases of primary congenital glaucoma are sporadic.
Which of the following statements concerning infantile glaucoma is FALSE?
ruptures of Descemet's membrane with infantile glaucoma are usually vertical, while those
associated with birth trauma are usually horizontal
offspring of an affected individual have a 5% chance of developing the condition
two-thirds of cases are male
subsequent offspring of the parents of an affected child have a 5% chance of manifesting the
condition
two-thirds of cases are bilateral
Your answer was CORRECT
Explanation
Ruptures in Descemet's membrane associated with infantile glaucoma are usually horizontal,
while those associated with birth trauma are usually vertical. Other options are true.
Mnemonic:
Haab's striae = Horizontal = associated with infantile glaucoma
Vogt's striae = Vertical = Vagina, birth trauma
All are recognized complications of argon laser trabeculoplasty EXCEPT:
peripheral anterior synechae formation
hypotony
hyphaema
uveitis
Your answer was CORRECT
Explanation
Potential side-effects of ALT include:
spike in IOP
hyphaema
uveitis
peripheral anterior synechiae
corneal endothelial damage
Side-effects of pilocarpine include all EXCEPT:
blurred vision
brow ache
migraine
hypermetropic shift
Your answer was INCORRECT
Explanation
Miotic agents such as pilocarpine cause a myopic shift. Other options supplied are true.
Compared to trabeculectomy without mitomycin, the use of mitomycin intraoperatively during
trabeculectomy may be associated with:
more inflammation and a more vascular-appearing bleb
higher long-term risk of endophthalmitis
lower surgical success rate
higher risk of retinal detachment
Your answer was INCORRECT
Explanation
Mitomycin C is a potent antineoplastic agent that intercalates with DNA and prevents its
replication. It is toxic to fibroblasts and vascular endothelial cells and hence gives rise to diffuse,
thin, avascular blebs. Reports suggest that these thin blebs may carry a higher risk of
endophthalmitis than is associated with filtering surgery without the use of mitomycin C.
Melanomalytic glaucoma is caused by:
trabecular meshwork seeding with melanoma tumour cells
trabecular meshwork obstruction with macrophages
angle neovascularisation secondary to anterior segment melanoma
an idiopathic increase in trabecular pigmentation
Your answer was CORRECT
Explanation
Melanomalytic glaucoma results from blockage of the trabecular meshwork by macrophages that
have engulfed material released from an intra-ocular melanoma tumor.
Melanomas may elevate IOP by pigment dispersion, inflammation, haemorrhage, angle-closure,
neovascularization of the angle or melanomalytic glaucoma. The most common cause of
secondary glaucoma due to intra-ocular melanoma is neovascularisation.
All the following are true of laser iridotomy EXCEPT:
pre-treatment with apraclonidine prevents IOP spikes
post-laser topical steroids are standard
laser should be applied superiorly in the periphery
haemorrhage may be induced by pressure from the contact lens
Your answer was CORRECT
Explanation
Haemorrhage is common following peripheral iridotomy (about 50%) and is due to rupture of iris
vessels. Haemorrhage can be controlled by applying pressure using the contact lens. Other options
above are true.
All of the following are associated with pseudoexfoliation syndrome EXCEPT:
poor pupillary dilation
peripheral iris transillumination defects
Sampaolesi's line
Krukenberg spindle
20% to 60% lifetime incidence of glaucoma
Your answer was INCORRECT
Explanation
Transillumination defects in PXF are typically peri-pupillary not peripheral as seen in pigment
dispersion syndrome.
PXF syndrome is characterised by:
bilateral in 50%
associated with older age-group than POAG
Krukenberg spindle
a heavily pigmented trabecular meshwork
Sampaolesi's line
peri-pupillary transillumination defects
decreased corneal endothelial cell density
poor pupillary dilation
nuclear sclerotic cataract
zonular weakness
lens subluxation
open angle glaucoma
Note: While Krukenberg spindle and Sampaolesi line are more commonly associated with
pigment dispersion syndrome, they are both recognised features of pseudo-exfoliation syndrome.
There are many sources to confirm this, e.g. Ariga M et al. Pseudoexfoliation Syndrome. J Curr
Glaucoma Pract. 2013 Sep-Dec; 7(3): 118–120.
The correct spot size for argon laser trabeculoplasty (ALT) is:
250 um
100 um
500 um
50 um
200 um
Your answer was CORRECT
Explanation
ALT uses a 50-micron beam with variable power to produce blanching or a tiny bubble at the
anterior pigmented edge of the trabecular meshwork. Typically 180 degrees is treated at one
sitting. Outflow facility typically improves following successful ALT.
A patient presents for review 1 month post-trabeculectomy for primary open angle glaucoma. On
examination, IOP is 4mmHG and the anterior chamber is shallow. The bleb is cystic and
avascular. The Seidel test shows sweating.
What treatment option is MOST likely to be effective:
bleb needling
autologous blood injection
oral acetazolamide
5-FU injection
Your answer was INCORRECT
Explanation
This patient has late bleb leakage. This is managed initially as for early overfiltration, including
aqueous suppressants and conjunctival tamponade (eg bandage contact lens) but these measures
are rarely effective. Other treatment depends on if there is 'sweating' or a true conjunctival
buttonhole. Sweating is treated by autologous blood injection and compression sutures. Button-
holes require revisional surgery such as conjunctival advancement or conjunctival autograft.
A patient presents with bilateral glaucoma. Gonioscopy reveals an anteriorly displaced, prominent
Schwalbe line with attached iris processes. The angle is otherwise open.
This clinical picture is most consistent with:
Peters' anomaly
iridocorneal endothelial (ICE) syndrome
posterior embryotoxon
Axenfeld-Rieger syndrome
Your answer was INCORRECT
Explanation
Figure: Axenfeld-Reiger syndrome
Axenfeld-Rieger syndrome is a spectrum of anterior segment defects characterised by an
anteriorly displaced Schwalbe line (posterior embryotoxon) and attached iris processes. Patients
may have glaucoma, developmental defects of facial bones and teeth. It is most commonly
inherited in an autosomal dominant pattern.
In the past, there were rigid definitions:
Axenfeld's anomaly was posterior embryotoxon and attached iris processes.
Rieger's anomaly was the components of Axenfeld's anomaly plus atrophy of the iris stroma.
Rieger's syndrome was Rieger's anomaly with skeletal, dental, or craniofacial abnormalities.
Note, however, that despite the above definitions, nowadays the term Axenfeld-Rieger syndrome is
considered more appropriate given the high incidence of overlapping findings, rather than rigidly
defining each syndrome/anomaly.
Which test may be useful in a patient with bilaterally narrow anterior chamber angles and normal
intraocular pressure?
edrophonium test
the prone-dark room test
topical steroid challenge
dexamethasone suppression test
thymoxamine test
Your answer was INCORRECT
Explanation
Angle-closure develops in only a small number of patients with narrow anterior chambers. A
number of provocative tests exist to attempt to cause angle closure in susceptible patients. Perhaps
the most predictive is the prone-darkroom test. Intraocular pressure is measured before and after
30 to 60 minutes of total dark adaptation attained with the patient prone. Darkness will induce
pupillary dilation, and prone positioning will move the lens forward. Both tend to increase
pupillary block.
Which of the following is LEAST likely to be found in a 9-month-old recently diagnosed with
primary congenital glaucoma?
open angle with high iris insertion on gonioscopy
cup to disc ratio of 0.4
corneal diameter of 10.0 mm
IOP of 23 mmHg
Your answer was CORRECT
Explanation
Normal corneal diameter at birth is 9.5 mm to 10.5 mm, growing to 10.0 mm to 11.5 mm at 1 year
of age. All forms of childhood glaucoma cause enlarged corneal diameter because of stretching of
immature collagen, with a corneal diameter 1 mm above average suspiscious and a diameter over
13 mm highly suspiscious for glaucoma.
Congenital glaucoma has an incidence of 1 in 12,500 births. It is bilateral in two-thirds of patients;
two-thirds are male; and about 10% of congenital glaucoma is familial (autosomal recessive).
Symptoms include tearing, photophobia, and blepharospasm.
Signs of congenital glaucoma include:
IOP over 21 mmHg
cup:disc ratio greater than 0.3 (present in only 2.6% of normal newborns)
horizontal corneal diameter greater than 12.5 mm with or without corneal oedema or breaks in
Descemet's membrane
open angle with anterior iris insertion - either flat insertion into the trabecular meshwork or,
less commonly, concave insertion with the plane of the iris posterior to scleral spur and
anterior iris stroma sweeping upward and inserting into the meshwork
no iris abnormality other than that previously described (i.e., no hypoplasia or corectopia)
Important diagnoses to exclude:
neural crest dysgenesis (aniridia, Peter's anomaly, Axenfeld-Rieger)
phakomatoses
metabolic abnormalities (Lowe's, homocystinuria, mucopolysaccharidoses)
inflammatory conditions (congenital rubella, herpes simplex iridocyclitis)
neoplasms (retinoblastoma, juvenile xanthogranuloma)
congenital diseases (X-linked megalocornea, Down syndrome, Patau's syndrome, Zellweger
syndrome, Rubinstein-Taybi syndrome, PHPV, retinopathy of prematurity)
trauma
steroid use
Which statement is TRUE regarding glaucoma management:
the Tonopen is more accurate than the Goldmann for IOP readings post-LASIK
direct-acting parasympathetic agents are more likely to cause cataract, punctal stenosis and retinal
tears compared to indirect-acting agents
intravenous urea has a greater IOP-lowering effect than intravenous mannitol but its routine
clinical use is limited by tissue toxicity
osmotic agents are more effective in inflamed eyes
Your answer was INCORRECT
Explanation
The Tonopen appears to have more accuracy for patients post-laser-assisted in situ keratomileusis
(LASIK) surgery compared to the Goldmann, where readings are artificially low after LASIK.
Indirect-acting parasympathomimetic agents tend to have the most pronounced systemic and
ocular side effects compared to direct-acting agents.
Intravenous mannitol is more effective than intravenous urea in lowering IOP. Mannitol is
distributed only in the blood compartment, whereas urea moves freely in total body water. As a
result, mannitol can generate a greater osmotic gradient than urea because its intravascular
concentration remains greater. The prevalence of intravenous urea as a hyperosmotic agent to
control IOP is decreasing in part because of its risk for tissue necrosis with extravasation.
In inflamed eyes, the integrity of the blood-aqueous barrier is compromised and this prevents
build-up of an osmotic gradient with agents such as mannitol, thus lowering their efficacy.
All of the following are ocular side-effects of topiramate EXCEPT:
suprachoroidal effusion
scleritis
myopia
blurred vision
open-angle glaucoma
Your answer was CORRECT
Explanation
Topiramate is associated with acute secondary angle-closure glaucoma. Most cases are bilateral,
and can lead to blindness if not treated or recognised. All ocular side-effects are reversible if
recognised early and the drug is discontinued.
Ocular side-effects of topiramate:
blurred vision
myopia
suprachoroidal effusion
angle-closure glaucoma
scleritis
Initial treatment of primary congenital glaucoma is usually:
trabeculotomy
goniotomy
trabeculectomy with augmentation
laser trabeculoplasty
Your answer was CORRECT
Explanation
Initial treatment of primary congenital glaucoma is goniotomy, which is usually performed at the
time of EUA (examination under anaesthesia). It involves making a horizontal incision at the
midpoint of the superficial layers of the trabecular meshwork. The success rate is 85%. Results are
poor if the corneal diameter is 14mm or more due to obliteration of the canal of Schlemm in such
eyes.
The most common mechanism of angle-closure glaucoma is:
pupillary block
plateau configuration
inflammatory synechial closure
angle rubeosis
Your answer was CORRECT
Explanation
Pupillary block refers to the obstruction of flow of aqueous from the posterior chamber to the
anterior chamber caused by a functional block between the lens and iris. It is the most frequent
cause of angle-closure glaucoma.
Gonioscopy in iridoschisis is likely to reveal:
narrow occludable angles
angle recession
angle neovascularization
angle hyperpigmentation
Your answer was INCORRECT
Explanation
Iridoschisis is thought to result from intermittent angle-closure, which causes iris atrophy and
schisis due to the high pressure. Findings include shallow anterior chamber with occludable angles
on gonioscopy. Iridoschisis usually involves the inferior iris.
Applanation tonometry has a tendency to produce falsely low readings under all of the following
conditions EXCEPT:
greater than 3 dioptres of with-the-rule astigmatism
high myopia
decreased central corneal thickness
excessive fluorescein
Your answer was CORRECT
Explanation
If the fluorescein rings are too narrow (too little fluoroscein), the IOP is underestimated; if they
are too thick (too much fluoroscein), the IOP is overestimated.
Goldmann tonometry may be falsely low in:
corneal oedema
low central corneal thickness
post laser refractive surgery
too little fluorescein dye
with-the-rule astigmatism over 3D
high myopia (due to decreased scleral rigidity)
Goldmann tonometry may be falsely high in:
increased central corneal thickness
digital pressure on the globe
applanation over a corneal scar
too much flurorescein drops
against-the-rule astigmatism over 3D
Iridocorneal endothelial syndrome has a correlation with:
herpes zoster virus
adenovirus
cytomegalovirus
herpes simplex virus
Your answer was CORRECT
Explanation
ICE syndrome is caused by an abnormal corneal endothelial cell layer which proliferates and
migrates across the angle and onto the iris. Glaucoma occurs in about 50% of cases, due to
synechial angle closure secondary to contraction of this abnormal tissue. PCR suggests a possible
herpes simplex viral origin to the disease process.
Glaucoma secondary to choroidal melanoma is usually the result of:
chronic low-grade trabeculitis affecting meshwork drainage
neovascular glaucoma
mechanical obstruction of the trabecular meshwork by tumour cells
angle invasion by solid metastases
Your answer was CORRECT
Explanation
Neovascular glaucoma is the most common mechanism of raised intraocular pressure in eyes with
choroidal melanoma or retinoblastoma. In ciliary body melanoma, anterior displacement of the
iris-lens diaphragm resulting in angle-closure is the most common mechanism. Iris melanomas
raise IOP by mechanical obstruction of the meshwork, either by direct angle invasion, or by
tumour cell-macrophage obstruction
All the following can occur in primary congenital glaucoma EXCEPT:
lens subluxation
scleral thinning
epithelial and stromal corneal oedema
anterior chamber deepening
narrowing of the scleral canal
Your answer was INCORRECT
Explanation
The scleral canal enlarges as part of the generalised enlargement of the globe in PCG. Together
with optic nerve damage from raised IOP, this scleral canal enlargement contributes to a larger
cup:disc ratio in PCG of greater than 0.3 - which is uncommon in normal infants. Other options
supplied above are true.
Iridoschisis is typically found in which quadrant of the iris:
temporally
inferiorly
superiorly
nasally
Your answer was CORRECT
Explanation
Iridoschisis is thought to result from intermittent angle-closure, which causes iris atrophy and
schisis due to the high pressure. Findings include shallow anterior chamber with occludable angles
on gonioscopy. Iridoschisis usually involves the inferior iris. Initial treatment is with peripheral
iridotomy.
Which of the following is NOT considered a potential side-effect of the hyperosmotic agents?
subdural hemorrhage
bronchoconstriction
mental confusion
congestive heart failure
Your answer was CORRECT
Explanation
Side-effects of hyperosmotic agents:
congestive heart failure
backache
headache
mental confusion
subdural, even subarachnoid, hemorrhages
Which one of the following statements is FALSE concerning pseudoexfoliation:
lens extraction alleviates the condition
the IOP is often higher than in POAG
it has a worse prognosis than POAG
it may be unilateral or bilateral
Your answer was CORRECT
Explanation
In pseudoexfoliation (PXF) lens extraction is not curative, although the pressure may be lowered
by a few mm of Hg. It is rare that cataract surgery in eyes with PXF would alleviate the condition
to the extent that pressure-lowering drops could be reduced or stopped.
Which visual field defect on Humphrey perimetry corresponds to the greatest glaucomatous optic
nerve damage?
paracentral scotoma
superior altitudinal defect
generalised depression
inferior quadrantanopia
Your answer was INCORRECT
Explanation
Generalised depression can occur from many non-glaucoma causes such as cataract. An inferior
quadrantanopia is a neurological field defect corresponding to a lesion in the parietal lobe. A
superior altitudinal defect in the context of glaucoma represents a substantial loss of optic nerve
function, and one that encroaches near to fixation. These patients would benefit from reducting
their IOP as low as possible.
All of the following are more common in normal tension glaucoma than primary open angle
glaucoma EXCEPT:
migraine
paraproteinamia
Raynaud's syndrome
serum auto-antbodies
systemic hypertension
Your answer was INCORRECT
Explanation
Nocturnal systemic hypotension is more common in NTG - not hypertensi
In the normal optic nerve head which rim is the thinnest:
nasal
temporal
inferior
superior
Your answer was CORRECT
Explanation
The normal healthy optic nerve rim has a characteristic configuration with the inferior rim
broadest followed by the superior, nasal and temporal rims. This is remembered by the pneumonic
the ISNT rule.
All are true of optic disc haemorrhages EXCEPT:
they are more common in normal tension glaucoma than pseudoexfoliation
they are flame-shaped
they occur in the nerve fibre layer
they are most common infero-nasally
Your answer was INCORRECT
Explanation
Optic disc haemorrhages occur most commonly infero-temporally.
A patient has short stature, aortic stenosis and mental retardation. On examination the discs are
cupped consistent with glaucoma. What is the most likely diagnosis?
Peter’s anomaly
Rieger’s syndrome
ICE syndrome
posterior embryotoxon
Your answer was CORRECT
Explanation
Rieger’s syndrome involves posterior embryotoxon (anteriorly displaced Schwalbe’s line with
50% risk of glaucoma) together with systemic features that may include:
craniofacial dysmorphism: such as hypertelorism, telecanthus, maxillary hypoplasia, and a
broad, flat nasal bridge
dental abnormalities: including microdontia, oligodontia, or hypodontia
systemic manifestations: including hypospadias, anal stenosis, pituitary abnormalities, growth
retardation, and cardiac valvular abnormalities.
Peter’s anomaly can also be associated with systemic abnormalities including developmental
delay, heart defects, hearing loss, CNS defects, gastrointestinal and genitourial defects. However,
Peter’s is characterized by a central corneal opacity which is inconsistent with the patient
described above where fundal views of the disc are possible.
This question came in the 2014 FRCOphth.
Which of the following meet Anderson's criteria for glaucomatous field damage on a Humphrey
field test:
1. CPSD that occurs in less than 5% of normal individuals on 2 occasions
2. a glaucoma hemifield test outside normal limits on two consecutive occasions
3. a cluster of three or more non-edge points, all depressed on pattern standard deviation at the
level of P < 5% on two consecutive occasions
3 only
1 only
1 and 2
1, 2 and 3
Your answer was INCORRECT
Explanation
Anderson's minimal criteria for glaucomatous damage include:
Glaucoma hemifield test outside normal limits on at least two consecutive occasions
A cluster of three or more non-edge points, all depressed on pattern standard deviation at the
level of P less than 5% one of which is at P less than 1%, on two consecutive occasions
CPSD that occurs in less than 5% of normal individuals on two consecutive occasions
Bimatoprost can be best described as a:
parasympathomimetic
synthetic prostamide analogue
cholinesterase inhibitor
F2-alpha prostaglandin analogue
Your answer was INCORRECT
Explanation
Bimatoprost is a synthetic prostamide analogue while travoprost, latanoprost and taflupost are F2-
alpha prostaglandin analogues.
Which one of the following conditions does NOT have the same pathogenesis of glaucoma as the
others?
Sturge-Weber syndrome
Carotid-cavernous sinus fistula
Thyroid eye disease
Aniridia
Your answer was INCORRECT
Explanation
The glaucoma associated with aniridia is caused by congenital agenesis of the angle structures. All
the other options supplied cause raised IOP by increasing episcleral venous pressure.
List the following drops in order of their propensity to cause allergic conjunctivitis in a glaucoma
patient (most likely first):
[Link]
[Link]
[Link]
3>2>1
3>1>2
2>3>1
1>2>3
Your answer was INCORRECT
Explanation
Prostaglandin analogues more commonly produce conjunctival hyperemia than true allergic
conjunctivitis. From 1% to 5% of patients taking dorzolamide may have allergic symptoms. Up to
20% of patients taking brimonidine (Alphagan) may show such symptoms.
Glaucoma which manifests at age 39 months is classified as:
infantile glaucoma
true congenital glaucoma
late-onset congenital glaucoma
juvenile glaucoma
Your answer was INCORRECT
Explanation
Congenital glaucoma is classified as:
True congenital glaucoma (40%) which develops in utero
Infantile glaucoma (55%) which develops before the third birthday
Juvenile glaucoma (5%) develops after the third birthday but before the age of 16 years
Aqueous humour has an increased concentration of which component by comparison to plasma?
glucose
carbon dioxide
ascorbate
protein
Your answer was CORRECT
Explanation
Compared with plasma, aqueous is slightly hypertonic and acidic. Aqueous has a marked excess
of ascorbate (15 times greater than that of arterial plasma) and a marked deficit of protein (0.2% in
aqueous as compared to 7% in plasma).
Regarding surgical trabeculectomy in inflammatory glaucoma all are true EXCEPT:
combined cataract and glaucoma surgery is recommended to avoid a 2nd operation
antimetabolites are recommended due to the high risk of failure
uveitis should be controlled for a minimum of 3 months pre-operatively
post-operative hypotony is more common than POAG
Your answer was INCORRECT
Explanation
Combined cataract and glaucoma surgery should be avoided with cataract surgery delayed until
about 6 months after trabeculectomy, as the results are better. This is due to the fact that the
cataract surgery, if done at the time of the trabeculectomy, encourages a further inflammatory
response. All other options above are true.
The most common mechanism of angle-closure glaucoma is:
pupillary block
angle rubeosis
inflammatory synechial closure
plateau configuration
Your answer was CORRECT
Explanation
Pupillary block refers to the obstruction of flow of aqueous from the posterior chamber to the
anterior chamber caused by a functional block between the lens and iris. It is the most frequent
cause of angle-closure glaucoma.
Which of the following may be accompanied by a falsely low Goldmann IOP measurement?
severe corneal scarring
too much fluorescein instilled
a history of refractive surgery
increased corneal thickness
Your answer was CORRECT
Explanation
Goldmann tonometry may be falsely low in:
corneal oedema
low central corneal thickness
post laser refractive surgery
too little fluorescein dye
with-the-rule astigmatism over 3D
high myopia (due to decreased scleral rigidity)
Goldmann tonometry may be falsely high in:
increased central corneal thickness
digital pressure on the globe
applanation over a corneal scar
too much flurorescein drops
against-the-rule astigmatism over 3D
A 72-year-old Caucasian man underwent intracapsular cataract extraction 25 years ago.
Approximately 15 years ago, he underwent secondary anterior chamber lens placement. Two years
ago, this eye underwent penetrating keratoplasty. Over the past year, he has had recurrent uveitis
in this eye, and on referral, his IOP is 25 mmHg. Gonioscopy reveals an open angle and a small
hyphaema.
The MOST appropriate management of his eye would be:
surgical removal of his anterior chamber lens
atropine
panretinal photocoagulation
topical corticosteroids
Your answer was INCORRECT
Explanation
This case is characteristic of the uveitis-glaucoma-hyphaema syndrome, which is caused by an ill-
fitting anterior chamber IOL. Modern flexible-haptic anterior chamber lenses are better tolerated
and less apt to cause these complications. Therefore this patient's anterior chamber IOL should be
replaced or, at least, removed.
All the following are true of chronic angle closure glaucoma EXCEPT:
Type 1 is caused by gradual, progressive synechial angle closure
the optic disc demonstrates cupping
Type 2 is caused by synechial closure from intermittent angle closure attacks
treatment of cataract is contraindicated when IOP is poorly controlled
Your answer was CORRECT
Explanation
Treatment of cataract is often beneficial in controlling IOP in chronic angle closure glaucoma, by
deepening the anterior chamber and opening the irido-corneal angle. Other options are true.
All of the following are typical findings in pseudo-exfoliation EXCEPT:
corneal endothelial pigment deposits
iris transillumination peripherally
anterior chamber flare
trabecular hyperpigmentation
Your answer was CORRECT
Explanation
Iris transillumination defects are found in pseudo-exfoliation but they typically occur around the
pupillary sphincter, with a classic moth-eaten appearance
All the following are true of laser iridotomy EXCEPT:
pre-treatment with apraclonidine prevents IOP spikes
laser should be applied superiorly in the periphery
haemorrhage may be induced by pressure from the contact lens
post-laser topical steroids are standard
Your answer was CORRECT
Explanation
Haemorrhage is common following peripheral iridotomy (about 50%) and is due to rupture of iris
vessels. Haemorrhage can be controlled by applying pressure using the contact lens. Other options
above are true.
Glaucoma which manifests at age 39 months is classified as:
infantile glaucoma
true congenital glaucoma
late-onset congenital glaucoma
juvenile glaucoma
Your answer was INCORRECT
Explanation
Congenital glaucoma is classified as:
True congenital glaucoma (40%) which develops in utero
Infantile glaucoma (55%) which develops before the third birthday
Juvenile glaucoma (5%) develops after the third birthday but before the age of 16 years
What is the most commonly encountered side-effect of carbonic anhydrase inhibitor
administration:
anaemia
kidney stones
paraesthesias
hypocalcaemia
gastrointestinal distress
Your answer was INCORRECT
Explanation
Paraesthesias are reported by most patients taking CA-inhibitors.
Changes in urine pH secondary to carbonic anhydrase inhibitors can predispose a patient to
calcium oxalate and calcium phosphate nephrolithiasis.
Aplastic anemia is a rare but potentially lethal side-effect related to the sulfa derivation of the
drugs.
Gastrointestinal distress may occur with CA-inhibitors but is not as common as paraesthesias.
Hypokaelemia may occur as a result of the effects on renal ion transport (but hypocalcaemia is not
seen).
Which sub-type of glaucomatous disc damage is associated with a temporal crescent?
senile sclerotic
concentrically enlarging
myopic
focal ischaemic
Your answer was CORRECT
Explanation
The sub-types of glaucomatous optic nerve damage are:
Type 1 (focal ischaemic): characterized by a focal notch in the neuroretinal rim. It usually
occurs in females with a history of migraine or vasospasm. It is typically associated with a
localised field defect near to fixation.
Type 2 (myopic glaucomatous): associated with a temporal crescent in the absence of
degenerative myopia. It is associated with dense superior or inferior field loss and tends to
occur in younger male patients.
Type 3 (senile sclerotic): characterised by a shallow saucerised cup and a gently sloping rim.
As its name suggests, it typically occurs in elderly patients and is associated with hypertension
and cardiovascular disease.
Type 4 (concentrically enlarging): characterised by thinning of the entire rim in the absence of
notching. It is associated with a high IOP at presentation and tends to occur in younger paints.
Field loss is frequently diffuse.
Type 5: mixed
All of the following may contribute to a shallow anterior chamber causing chronic angle-closure
glaucoma EXCEPT:
reverse pupil block
seclusio pupillae
iris bombe
mature lens
hyperopia
Your answer was CORRECT
Explanation
Reverse pupil block (where the anterior chamber pressure is higher than the posterior chamber
pressure) is a theoretical cause of concave iris configuration seen in pigment dispersion syndrome
in myopes. It has no role in shallow anterior chambers and angle-closure glaucoma.
A patient presents 1-day post trabeculectomy with a shallow anterior chamber. The intra-ocular
pressure is 2mmHg. The iridotomy is patent, the bleb is flat.
What is the MOST likely diagnosis:
scleral flap leak
aqueous misdirection syndrome
conjunctival bleb leak
blebitis
Your answer was INCORRECT
Explanation
A shallow anterior chamber with high IOP post-trabeculectomy can be the result of:
Pupillary block: this is due to a non-patent peripheral iridotomy with iris bombe. The bleb is
flat and Siedl's is negative
Aqueous misdirection: this is due to aqueous being directed posteriorly into the vitreous. The
AC is shallow, the IOP high, the bleb flat and Seidel negative. However, the iridotomy in this
case is patent with no iris bombe.
A shallow anterior chamber with low IOP post-trabeculectomy can be the result of:
Scleral flap leak: the bleb is well formed and Seidel test negative
Conjunctival bleb leak: the bleb is flat and Seidel test positive
What percentage of blind registrations are due to glaucoma in Western countries?
10%
30%
60%
1%
Your answer was CORRECT
Explanation
Glaucoma is responsible for approximately 10% of blind registrations in Europe and the USA.
Which of the following is FALSE regarding the epidemiology of primary angle closure glaucoma:
females are more commonly affected than males
it is more common in South-East Asians and Chinese than in Caucasians
it is more common in blacks than in whites
it is more likely in older patients
Your answer was INCORRECT
Explanation
Primary angle closure is least common in blacks and most common in South-East Asians, Chinese
and Eskimos. Other options above are true.
Glaucoma is more frequent in Sturge-Weber patients with facial haemangioma involving:
the tip of the nose
the maxillary distribution of the trigeminal nerve
the ophthalmic distribution of the trigeminal nerve
the upper eye lid
Your answer was CORRECT
Explanation
Glaucoma is much more common in Sturge-Weber patients with facial haemangioma involving
the upper eye lid.
All of the following examination findings are consistent with phacolytic glaucoma EXCEPT:
corneal oedema
closed irido-corneal angle
pseudo-hypopyon
hypermature cataract
Your answer was CORRECT
Explanation
Phacolytic glaucoma is caused by high molecular weight lens proteins which have leaked through
an intact capsule of a hypermature cataract and have blocked the trabecular meshwork. Findings
include an open irido-corneal angle, corneal oedema and pseudo-hypopyon (lens material and
macrophages).
Phacolytic glaucoma is not to be confused with phacoanaphylactic (phaco-antigenic) glaucoma
which is caused by an auto-immune granulomatous reaction to lens proteins in an eye with a
ruptured capsule.
Phacomorphic glaucoma is caused by secondary angle closure due to an intumescent lens and will
demonstrate closed irido-corneal angles on gonioscopy.
A 60-year-old lady presents with moderate corneal oedema, cells 1+ in the anterior chamber and
IOP of 34 mmHg. 2 months ago he was treated for herpes zoster ophthalmicus and he is on
amitryptyline for trigeminal neuralgia.
What is the most appropriate anti-glaucoma treatment?
timolol
pilocarpine
brinzolamide
brimonidine
bimatroprost
Your answer was CORRECT
Explanation
Timolol is the most appropriate drop in this case.
Prostaglandin analogues may increase inflammation and can cause cystoid macular oedema so are
not traditionally used in the context of uveitis.
Pilocarpine likewise can lead to an increase in permeability of the blood aqueous barrier and
causes miosis (which can lead to seclusio pupillae) so it is also not recommended with uveitis.
Carbonic anhydrase inhibitors can exacerbate corneal oedema due to their effect on the endothelial
pump mechanism.
Alpha agonists such as brimonidine can precipitate a hypertensive crisis with tricyclic
antidepressants.
All of the following are side-effects of topical beta-adrenergic antagonists EXCEPT:
weakened myocardial contractility
increased plasma high-density lipoprotein cholesterol levels
exercise intolerance
airway obstruction
Your answer was CORRECT
Explanation
Blockade of beta-1-adrenergic receptors:
slows the pulse rate
weakens myocardial contractility
reduced exercise tolerance
Blockade of beta-2-adrenergic receptors produces:
bronchoconstriction
peripheral arterial constriction
aqueous suppression of ciliary epithelium
Topical timolol has been shown to decrease plasma high-density lipoprotein cholesterol levels.
Carteolol is a topical non-selective beta-adrenergic antagonist that has intrinsic sympathomimetic
activity and has been shown to have less adverse effect on plasma lipids. It may also cause less
cardiovascular effects such as bradycardia.
Betaxolol is relatively beta-1-selective and may have less of an effect on the respiratory muscles
Choose the most accurate statement. Phacolytic glaucoma is caused by:
secondary angle closure due to an intumescent lens
high molecular weight lens proteins which have leaked through an intact capsule
the release of inflammatory cytokines after cataract surgery in a mature lens
an auto-immune granulomatous reaction to lens proteins in an eye with a ruptured capsule
Your answer was CORRECT
Explanation
Phacolytic glaucoma is caused by high molecular weight lens proteins which have leaked through
an intact capsule and have blocked the trabecular meshwork. This is not to be confused with
phacoanaphylactic (phaco-antigenic) glaucoma which is caused by an auto-immune
granulomatous reaction to lens proteins in an eye with a ruptured capsule. Phacomorphic
glaucoma is caused by secondary angle closure due to an intumescent lens.
All are true of pilocarpine EXCEPT:
its IOP lowering effect is additive with prostaglandin analogues
pilocarpine gel at night is known to induce diffuse corneal haze
it increases trabecular outflow
it causes myopic shift
Your answer was INCORRECT
Explanation
The IOP lowering effect of pilocarpine is additive (complimentary to) beta-blockers, but not with
prostaglandin analogues.
Which of the following is considered the MAIN advantage of non-penetrating glaucoma surgery
compared to traditional trabeculectomy?
lower incidence of post-operative complications
technically easier
less time-consuming
better IOP reduction when compared to standard trabeculectomy
Your answer was CORRECT
Explanation
Non-penetrating glaucoma surgery includes deep sclerectomy with collagen implant and deep
sclerectomy with injection of viscoelastic into Schlemm's canal (viscocanalostomy). The surgery
involves creating a superficial scleral flap and a deeper scleral dissection underneath to leave
behind a thin layer of sclera and Descemet's membrane. Preliminary data comparing non-
penetrating procedures to standard trabeculectomy shows better IOP reduction after standard
trabeculectomy but a lower incidence of post-operative complications such as hypotony after non-
penetrating procedures. However, the non-penetrating surgeries are technically more difficult.
What treatment is MOST likely to be effective in a patient with an encapsulated trabeculectomy
bleb?
subconjunctival injection of 5-FU
bleb needling
suture release
ocular massage
Your answer was INCORRECT
Explanation
All the options above are treatments for filtration failure following trabeculectomy. The choice of
treatment initiated depends on the cause of filtration failure. For an encapsulated bleb, bleb
needling is most likely to be effective, as this breaks the tenon cyst. For episcleral fibrosis, 5-FU
injection is most appropriate to reduce the drive for fibrosis. For scleral flap over-tight sutures,
suture release plus or minus ocular massage is effective.
Which of the following topical steroids will likely cause the highest rise in intraocular pressure?
hydrocortisone 1%
prednisolone 1%
medrysone 1%
dexamethasone 0.1%
fluorometholone 0.1%
Your answer was INCORRECT
Explanation
Compared to the other medications listed, dexamethasone 0.1% is associated with the highest rise
in intraocular pressure and is the most potent steroid of those listed.
Which of the following is a direct lens for gonioscopy?
Koeppe
Zeiss
Sussman
Goldmann
Posner
Your answer was INCORRECT
Explanation
Koeppe, Richardson, Barkan, Wurst and Swan-Jacob lenses are used in direct gonioscopy, while
the Goldmann, Zeiss, Posner and Sussman lenses are used in indirect gonioscopy.
Direct gonioscopy is useful during intraocular surgery e.g. goniotomy.
A 74-year-old lady presents with unilateral phacodonesis, pigmented lines anterior to the
trabecular meshwork and an IOP of 33mmHg.
What is the most likely diagnosis?
glaucoma capsulare
exfoliation syndrome
pigmentary dispersion
iridocorneal endothelial syndrome
Your answer was INCORRECT
Explanation
Glaucoma capsulare is another name for pseudoexfoliation syndrome (PXF), not to be confused
with exfoliation syndrome and associated glass-blowers cataract.
A pigmented line (Sampaolesi’s line) may be seen in PXF and pigment dispersion. However, in
the latter, there should be other signs like Krukenberg’s spindle and patients usually present at a
younger age. Pigment dispersion also would not explain phacodonesis.
Which of the following investigations is MOST appropriate in a neonate presenting with
glaucoma:
urinalysis for proteinuria and aminoaciduria
serum galactose levels
serum phytanic acid levels
serum anti-rubella immunoglobulin IgG
Your answer was INCORRECT
Explanation
Rubella immunoglobulin IgM titers might be of value in a neonate, but IgG titers are not
diagnostic because they are of maternal origin during the first 3 to 6 months of life. Glaucoma is
not a feature of galactosaemia or Refsum's disease. Lowe's syndrome must be considered, hence
the utility of urinalysis.
In which condition is trabeculectomy failure LEAST likely:
pseudoexfoliation glaucoma
inflammatory glaucoma
neovascular glaucoma
irido-corneal endothelial syndrome glaucoma
Your answer was CORRECT
Explanation
Risk factors for trabeculectomy failure include:
neovascular glaucoma
inflammatory glaucoma
ICE syndrome glaucoma
angle-recession glaucoma
previous failed trabeculectomy
previous conjunctival or cataract surgery
longterm glaucoma drop usage
black patients
patients under 40 years
Which of the following topical glaucoma medications used unilaterally can cause a macroscopic
appearance similar to Fuch's iridocyclitis?
dipivefrin
brimonidine
dorzolamide
latanoprost
Your answer was CORRECT
Explanation
Prostaglandin analogues have several pro-inflammatory side-effects, including inciting
development of an anterior uveitis and cystoid macular oedema. Unilateral use can also result in
changes to iris pigmentation with resulting heterochromia (though prostaglandins increase iris
pigmentation, while Fuch's typically causes lightening of iris pigmentation).
All are true of topical beta-blockers EXCEPT:
their IOP-lowering effect is less during sleep
they act primarily by reducing aqueous secretion
there is an additional IOP-lowering effect if used with a systemic beta-blocker
IOP reduction is better in combination with prostaglandins than CA-inhibitors
Your answer was CORRECT
Explanation
In general, there is no additional IOP lowering effect if a topical beta blocker is used in a patient
already on a systemic beta-blocker. Other options above are true.
All are true of pigment dispersion syndrome (PDS) EXCEPT:
men with PDS are more likely to develop glaucoma than women
myopes with PDS are more likely to develop glaucoma than non-myopes
diagnosis is usually in the sixth to seventh decades
blacks are less likely to be affected than whites
Your answer was CORRECT
Explanation
Diagnosis of PDS is usually in the third or fourth decades. Other options are true.
All of the following are risk factors for developing glaucoma EXCEPT:
increased central corneal thickness
high intra-ocular pressure
parapapillary changes
high myopia
old age
Your answer was INCORRECT
Explanation
Decreased corneal thickness is a risk factor for glaucoma, because thin corneas tend to under-
estimate the actual intra-ocular pressure. Other options above are true.
The rate of production of aqueous humour is approximately:
2 millilitres per minute
3 microlitres per minute
30 microlitres per minute
3 millilitres per minute
Your answer was INCORRECT
Explanation
The rate of production of aqueous is between 2 and 4 microlitres per minute.
Which is FALSE regarding pseudoexfoliation glaucoma?
the prognosis is poorer than POAG
laser trabeculoplasty is often highly effective initially
it presents earlier in life than POAG
it is usually an open-angle glaucoma
Your answer was CORRECT
Explanation
Pseudo-exfoliation glaucoma presents later in life (in the seventh decade) than primary open angle
glaucoma. Other options are true.
Two months ago, a 78-year-old man suffered a central retinal vein occlusion in his left eye. He
presents with a red, painful eye and florid rubeosis with an IOP of 46 mmHg.
What is the most appropriate management?
laser iridotomy
diode cyclophotocoagulation
glaucoma tube implantation
panretinal photocoagulation
Your answer was CORRECT
Explanation
The definitive treatment for neovascular glaucoma is to destroy areas of ischaemic retina in the
eye by panretinal photocoagulation or cryotherapy. Glaucoma tube implantation would be high-
risk in this hot, inflamed eye. Diode cyclophotocoagulation is not the first line of therapy for this
condition.
The second most common cause of neovascular glaucoma is:
central retinal artery occlusion
proliferative diabetic retinopathy
central retinal vein occlusion
longstanding retinal detachment
Your answer was CORRECT
Explanation
The most common cause of neovascular glaucoma is central retinal vein occlusion (36%) followed
by proliferative diabetic retinopathy (32%). Other causes are carotid occlusive disease, CRAO,
intra-ocular tumours, intra-ocular inflammation, or longstanding retinal detachments.
All are true of intermittent (sub-acute) angle closure EXCEPT:
attacks may be precipitated by emotional stress
it is characterised by a history of transient blurring of vision with haloes
the eye is usually red during an attack
treatment with peripheral iridotomy is recommended
attacks may be broken by sleep
Your answer was INCORRECT
Explanation
During an attack of sub-acute angle closure, vision is blurred with haloes around lights due to
transient cornea oedema. There may be associated ocular discomfort or headache, but the eye is
usually white. Attacks are recurrent and are usually broken after 1-2 hours by physiological miosis
(e.g. bright light or sleep). Treatment is with prophylactic peripheral laser iridotomy.
Glaucoma tube implant surgery is indicated in all of the following circumstances EXCEPT:
previously failed filtration surgery with anti-metabolites
congenital glaucoma patient with poor visual potential
glaucoma patient with previous vitrectomy with scleral buckle
uveitic glaucoma
Your answer was INCORRECT
Explanation
It may not be wise to perform incisional surgery in an eye with poor visual potential given the
risks and, at times, complicated postoperative course of tube shunt surgery. Tubes are usually
reserved for those glaucoma cases in which there is salvageable vision and standard filtration
surgery would fail or has already failed.
Thus glaucoma implant surgery is indicated in the following situations:
failed trabeculectomy
active uveitis
neovascular glaucoma
inadequate conjunctiva
impending need for PK
synechieal angle closure (e.g. ICE syndrome)
Note: in a patient with previous vitrectomy and a scleral buckle for retinal detachment, there is an
argument that with the buckle in situ, there is limited real estate to insert a plate. However, this is
only smoke and mirrors. With a previous retinal detachment, the scleral buckle is no longer
performing any useful job (which is complete once the retina re-attaches, within the first couple
months), so the buckle can be explanted with ease should it be interfering with the potential site of
a glaucoma plate.
Glaucoma tube implant surgery is indicated in all of the following circumstances EXCEPT:
previously failed filtration surgery with anti-metabolites
congenital glaucoma patient with poor visual potential
glaucoma patient with previous vitrectomy with scleral buckle
uveitic glaucoma
Your answer was INCORRECT
Explanation
It may not be wise to perform incisional surgery in an eye with poor visual potential given the
risks and, at times, complicated postoperative course of tube shunt surgery. Tubes are usually
reserved for those glaucoma cases in which there is salvageable vision and standard filtration
surgery would fail or has already failed.
Thus glaucoma implant surgery is indicated in the following situations:
failed trabeculectomy
active uveitis
neovascular glaucoma
inadequate conjunctiva
impending need for PK
synechieal angle closure (e.g. ICE syndrome)
Note: in a patient with previous vitrectomy and a scleral buckle for retinal detachment, there is an
argument that with the buckle in situ, there is limited real estate to insert a plate. However, this is
only smoke and mirrors. With a previous retinal detachment, the scleral buckle is no longer
performing any useful job (which is complete once the retina re-attaches, within the first couple
months), so the buckle can be explanted with ease should it be interfering with the potential site of
a glaucoma plate.
All of the following are true of parapapillary changes in glaucoma EXCEPT:
the alpha zone displays variable hyper- and hypo- pigmentation of the RPE
the alpha zone occurs more frequently in patients with POAG
in ocular hypertension the presence of parapapillary changes correlates with the subsequent
development of glaucoma
the beta zone exhibits chorioretinal atrophy with visibility of choroidal vessels
Your answer was CORRECT
Explanation
Parapapillary changes can be divided into an alpha and a beta zone.
The alpha zone (outer of the two) displays variable hyper- and hypo- pigmentation of the
RPE. The alpha zone occurs with the same frequency in normal subjects as in POAG, but the
area is larger in POAG.
The beta zone (inner of the two) represents loss of RPE and choriocapillaris leaving intact
choroidal vasculature. It occurs with greater frequency and extent in POAG compared to
normal.
Which statement about Goldmann applanation tonometry is FALSE?
it is less affected by ocular rigidity compared to Schiotz tonometry
it works on the basis of the Fick principle, which holds that the pressure inside an ideal sphere is
equal to the force required to flatten the sphere divided by the area of flattening
Goldmann tonometer tip alignment is important in accurately determining IOP in eyes with high
degrees of corneal astigmatism
IOP on the Goldmann scale is equal to the force required to flatten the cornea multiplied by a
factor of 10
the diameter of flattening, 4.06 mm, is based on counterbalancing the corneal resistance and
the capillary attraction of tears for the tonometer head
Your answer was CORRECT
Explanation
The diameter of flattening is 3.06mm, which is based on counterbalancing the corneal resistance
and the capillary attraction of tears for the tonometer head.
Unlike Schiotz tonometry, Goldmann applanation tonometry displaces a very small volume of
aqueous humor from the eye and does not significantly increase intraocular pressure. For this
reason, Goldmann measurements are essentially independent of ocular rigidity.
Which of the following agents should be used with caution in a patient with narrow angles?
bimatoprost
acetazolamide
timolol
brimonidine
Your answer was INCORRECT
Explanation
Acetazolamide and the sulphonamide derivatives have been described as precipitating on rare
occasions an idiosyncratic reaction with raised IOP due to anterior rotation of the ciliary body
with malignant glaucoma and shallowing of the anterior chamber. Acetazolamide is nonetheless
used in intractable narrow-angle glaucoma, but awareness of this reaction is important. The alpha-
2 agonists such as brimonidine and apraclonidine cause a mid-miosis in normal eyes, and
therefore are not a risk for precipitating an acute angle closure attack. Apraclonidine only causes
mydriasis in the context of Horner's syndrome but not a non-Horner's eye.
Which statement regarding secondary glaucoma is TRUE?
Fuchs iridocyclitis may cause angle neovascularisation resulting in synechial angle closure
after intra-ocular haemorrhage, haemolytic glaucoma occurs sooner than ghost cell
glaucoma
alkali burns are less likely to be associated with glaucoma than acid burns
ghost cell glaucoma is caused by haemoglobin-laden macropages blocking the meshwork
Your answer was INCORRECT
Explanation
Fuchs' heterochromatic iridocyclitis is considered a secondary open-angle glaucoma. The
glaucoma can be difficult to control and does not parallel the degree of inflammation. Rubeosis
can occur in this condition but it is odd in that the vessels are particularly prone to bleed, but do
not induce synechialisation.
In haemolytic glaucoma, haemoglobin-laden macrophages block the trabecular meshwork,
whereas in ghost cell glaucoma, rigid, degenerated khaki-colored red blood cells from the vitreous
enter the anterior chamber and obstruct the TM. Haemolytic glaucoma may occur within days of
hemorrhage, whereas ghost cell glaucoma is seen weeks to months later.
Alkali injury is more likely to cause glaucoma than acid, because it penetrates ocular structures
and causes more damage to trabecular meshwork structures.
Which of the following glaucoma medications is contraindicated in a patient with a history of
paroxysmal tachycardia?
timolol
pilocarpine
dipivefrin
acetazolamide
Your answer was INCORRECT
Explanation
Although their use has been in decline, both epinephrine and dipivefrin are beta-2 adrenergic
agonists with potentially serious systemic side effects. They are contraindicated in patients with
heart rhythm abnormalities. Timolol, as a beta antagonist, is not a cardiostimulant (quite the
opposite); neither is pilocarpine.
Which is FALSE of the epidemiology of normal tension glaucoma:
it occurs more frequently in Japan than in Europe
patients are more likely to have a family history of POAG than the normal population
females are at greater risk than males
patients tend to be younger than those with POAG
Your answer was INCORRECT
Explanation
Patients with NTG tend to be significantly older than those with POAG. Other options supplied
are true.
The distribution of intra-ocular pressure (IOP) in the population is:
linear
Gaussian with a skew to the right
Gaussian with a skew to the left
symmetrical Gaussian
Your answer was INCORRECT
Explanation
The distribution of IOP in the population is Gaussian with a skew to the right. The mean IOP is
16mmHg with a range of 11-21mmHg within 2 standard deviations of the mean.
All are true of pseudoexfoliation glaucoma by comparison to primary open-angle glaucoma
EXCEPT:
there is a greater initial response to laser therapy
it more often requires surgical intervention
it is a more symmetric disease
there is more trabecular pigmentation
Your answer was CORRECT
Explanation
In pseudoexfoliation, fibrillar material is deposited in the anterior segment of the eye. Patients are
often resistant to medical therapy, but laser trabeculoplasty is often very effective initially.
Pseudoexfoliation also differs from primary open-angle glaucoma in that it is often monocular or
asymmetric and has greater pigmentation of the trabecular meshwork, as well as pigment
deposited anterior to Schwalbe line (Sampaolesi line).
Which statement is FALSE regarding carbonic anhydrase inhibitors?
carbonic anhydrase inhibitors may cause idiosyncratic and transient acute myopia
aqueous production in the eye is not significantly reduced until more than 90% of the carbonic
anhydrase activity is inhibited
metabolic acidosis is greater with oral acetazolamide than with IV injection of acetazolamide
carbonic anhydrase inhibitors cause reduced excretion of urinary citrate or magnesium, therefore
predisposing to formation of kidney stones
Your answer was INCORRECT
Explanation
Metabolic acidosis is greater with IV injection of acetazolamide. Other statements are true.
All of the following are true of mitomycin C in trabeculectomy surgery EXCEPT:
it is more potent than 5-FU
it has an anti-fibroblast and anti-vascular effect
it is an alkylating agent
it is more likely to cause corneal epithelial defects than 5-FU
Your answer was INCORRECT
Explanation
MMC is an alkylating agent, with inhibitory effects on DNA and protein synthesis. It inhibits
fibroblast proliferation and vascular ingrowth. It is more potent than 5-FU and is used in high-risk
trabeculectomy cases. It may cause cystic thin-walled blebs and hypotony. It can cause corneal
epithelial defects, but these are more prominent with 5-FU.
Which molecule is most closely linked to the development of glaucoma:
platelet-derived growth factor
cortisol
IL-1
TNF-alpha
ELAM-1
Your answer was INCORRECT
Explanation
Glaucomas of diverse aetiology are linked by the presence of endothelial leukocyte adhesion
molecule 1 (ELAM-1), which indicates activation of a stress response in the trabecular meshwork
cells of eyes with glaucoma.
What is the best initial therapy for malignant glaucoma?
laser iridotomy
pilocarpine 2%
lens removal
mydriatic-cycloplegic therapy
Your answer was CORRECT
Explanation
The initial treatment of aqueous misdirection is with mydriatic-cycloplegic therapy in an attempt
to rotate the ciliary body and re-direct aqueous flow. If there is a poor response, treatment is
directed at disrupting the hyaloid face, either by a formal vitrectomy for phakic patients, or by a
YAG capsulotomy-hyaloidotomy for pseudophakic patients.
The agent most likely to cause topical sensitisation and medicamentosa is:
epinephrine
apraclonidine
betaxolol
pilocarpine
timolol
Your answer was CORRECT
Explanation
Epinephrine has a well-established tendency to provoke irritation and allergic responses. More
than one fifth of patients will eventually experience an adverse local reaction with prolonged use.
Which of the following clinical features is LEAST concerning for visual prognosis in a child
presenting with infantile glaucoma?
presence of an RAPD
onset less than 3 months of age
corneal diameter over 14 mm at the time of diagnosis
optic nerve cupping over 0.6
Your answer was INCORRECT
Explanation
Optic cupping is reversible in children. Afferent pupillary defects generally indicate irreversible
optic nerve damage.
What is the approximate prevalence of glaucoma in the population over the age of 40?
1 in 1000
1 in 10
1 in 10 000
1 in 100
Your answer was INCORRECT
Explanation
The prevalence of glaucoma is between 1-2% of the adult population over 40 years.
Which sub-type of glaucomatous disc damage is associated with younger age at presentation, high
IOP at presentation and diffuse field loss?
mixed
focal ischaemic
myopic glaucomatous
concentrically enlarging
Your answer was INCORRECT
Explanation
The sub-types of glaucomatous optic nerve damage are:
Type 1 (focal ischaemic): characterized by a focal notch in the neuroretinal rim. It usually
occurs in females with a history of migraine or vasospasm. It is typically associated with a
localised field defect near to fixation.
Type 2 (myopic glaucomatous): associated with a temporal crescent in the absence of
degenerative myopia. It is associated with dense superior or inferior field loss and tends to
occur in younger male patients.
Type 3 (senile sclerotic): characterised by a shallow saucerised cup and a gently sloping rim.
As its name suggests, it typically occurs in elderly patients and is associated with hypertension
and cardiovascular disease.
Type 4 (concentrically enlarging): characterised by thinning of the entire rim in the absence of
notching. It is associated with a high IOP at presentation and tends to occur in younger
patients. Field loss is frequently diffuse.
Type 5: mixed
Which is FALSE regarding plateau iris configuration?
the ciliary process is anteriorly positioned
it is more common in South-East Asians than whites
the iris plane is flat
the anterior chamber depth is shallow
Your answer was INCORRECT
Explanation
The anterior chamber depth is normal centrally in plateau iris syndrome. However, the irido-
corneal angle is shallow because the ciliary processes are anteriorly positioned. The iris plane is
flat as opposed to convex in usual narrow angles. It is more common in South-East Asians than
whites.
All of the following are true of uveitic glaucoma EXCEPT:
miotics are best avoided
treating the intraocular inflammation is as important as lowering IOP
prostaglandins should be used with caution
ALT may be a helpful adjunct if medications are ineffective
Your answer was INCORRECT
Explanation
Ocular inflammation can lead to glaucoma by a variety of mechanisms including:
obstruction of the trabecular meshwork by inflammatory debris
increased viscosity of aqueous humor
neovascularization
uveal effusion
pupillary block
formation of peripheral anterior synechiae
Treatment depends on the underlying condition, but in most cases, inflammation is suppressed by
some combination of topical, periocular, or oral corticosteroids. Occasionally, other
immunosuppressive agents are needed.
Elevated IOP is generally managed by topical and oral glaucoma medications as needed. Miotics
are usually avoided because they lead to increased pain and congestion and promote the formation
of posterior synechiae. Prostaglandin agents are used with caution because they may exacerbate
uveitis and CMO.
ALT is not very helpful in eyes with active inflammation.
All the following are characteristic of intermittent (sub-acute) angle closure EXCEPT:
narrow angles in between episodes
attacks precipitated by physiological mydriasis
attacks broken by sleep
attacks characterized by severe ocular pain and nausea
Your answer was CORRECT
Explanation
During an attack of sub-acute angle closure, vision is blurred with haloes around lights due to
transient cornea oedema. There may be associated ocular discomfort or headache, but the eye is
usually white. It is not typical for pain to be associated with nausea or vomiting in sub-acute angle
closure (though this may occur in acute congestive glaucoma). Attacks are recurrent and are
usually broken after 1-2 hours by physiological miosis (e.g. bright light or sleep). Treatment is
with prophylactic peripheral laser iridotomy.
A patient presents 1-day post trabeculectomy with a shallow anterior chamber. The intra-ocular
pressure is 2mmHg. The iridotomy is patent, the bleb is formed and Seidel test is negative.
What is the MOST likely diagnosis:
scleral flap leak
conjunctival bleb leak
blebitis
aqueous misdirection syndrome
Your answer was CORRECT
Explanation
A shallow anterior chamber with high IOP post-trabeculectomy can be the result of:
Pupillary block: this is due to a non-patent peripheral iridotomy with iris bombe. The bleb is
flat and Siedl's is negative
Aqueous misdirection: this is due to aqueous being directed posteriorly into the vitreous. The
AC is shallow, the IOP high, the bleb flat and Seidel negative. However, the iridotomy in this
case is patent with no iris bombe.
A shallow anterior chamber with low IOP post-trabeculectomy can be the result of:
Scleral flap leak: the bleb is well formed and Seidel test negative
Conjunctival bleb leak: the bleb is flat and Seidel test positive
A 35-year-old woman complains of decreased vision and glare in her right eye. On examination,
you notice right corneal oedema with mild iris atrophy and broad-based peripheral strands at the
irido-corneal angle. The IOP is 38 mmHg. Her left eye is normal.
What is the most likely glaucoma diagnosis?
pigment dispersion glaucoma
Chandler syndrome
post-trauma with angle recession glaucoma
iridoschisis glaucoma
Fuch's heterochromic iridocyclitis
Your answer was CORRECT
Explanation
This patient has iridocorneal endothelial syndrome (ICE), which is a condition caused by
abnormal proliferation and migration of corneal endothelial cells. These cells proliferate
throughout the iridocorneal angle (synechiae) and iris (pupil and iris anomalies) and lead to
corneal decompensation and secondary glaucoma that is difficult to control medically. ICE
syndrome was traditionally been described as three overlapping variants:
Progressive iris atrophy: iris changes including atrophy, corectopia and polycoria
Chandler syndrome: hammered-silver corneal endothelium and corneal oedema with varaible
iris stromal atrophy
Cogan-Reese syndrome: diffuse iris naevi
Which of the following complications of filtration surgery is more likely with an inferiorly-placed
bleb?
choroidal haemorrhage
cystic bleb encapsulation
cataract
endophthalmitis
aqueous misdirection syndrome
Your answer was INCORRECT
Explanation
Endophthalmitis is more common with inferiorly-placed blebs, probably due to the pooling of
infectious agents in the tear film inferiorly.
A 58-year-old man presents 4-days post-trabeculectomy. He complains of severe pain and
decreased vision in his operated eye. Examination reveals visual acuity 6/60, a diffusely shallow
anterior chamber, and no view of the posterior segment. There is no excessive anterior chamber
inflammation, and there are no vitreous cells observed.
These findings are in keeping with:
1. endophthalmitis
2. malignant glaucoma
3. suprachoroidal hemorrhage
2 only
1,2 and 3
1 only
2 and 3
1 and 3
Your answer was INCORRECT
Explanation
Post-trabeculectomy pain and decreased vision require evaluation for endophthalmitis, malignant
glaucoma, and suprachoroidal hemorrhage. However, the clear anterior chamber in this case
essentially excludes endophthalmitis. In suprachoroidal haemorrhage, there is often no fundal
view (as in this case) due to break-through vitreous haemorrhage; while in malignant glaucoma
there may be a limited view from corneal decompensation.
A patient presents for his 1-day post-trabeculectomy review. On examination, the anterior
chamber is very shallow with peripheral irido-corneal touch. The IOP is 50mm. The iridotomy is
patent. The bleb is flat and Seidel test is negative.
What is the most likely diagnosis:
scleral flap leakage
pupillary block
bleb leakage
aqueous misdirection syndrome
Your answer was CORRECT
Explanation
A shallow anterior chamber with high IOP post-trabeculectomy can be the result of:
Pupillary block: this is due to a non-patent peripheral iridotomy with iris bombe. The bleb is flat and
Siedl's is negative
Aqueous misdirection: this is due to aqueous being directed posteriorly into the vitreous. The AC is
shallow, the IOP high, the bleb flat and Seidel negative. However, the iridotomy in this case is
patent with no iris bombe.
A shallow anterior chamber with low IOP post-trabeculectomy can be the result of:
Scleral flap leak: the bleb is well formed and Seidel test negative
Conjunctival bleb leak: the bleb is flat and Seidel test positive
A patient presents for a 1-day post-trabeculectomy review. The anterior chamber is shallow, the
IOP is 2mmHg, Siedl's test is negative and the bleb is well formed.
Which treatment is LEAST likely to be effective:
oral acetazolamide
Nd:YAG laser iridotomy
pressure padding
topical atropine
Your answer was INCORRECT
Explanation
This patient has exuberant post-operative flap drainage. The initial treatment of over-filtration is
with topical atropine (to prevent pupil block) and aqueous suppressants such as topical beta
blockers and oral acetazolamide to temporarily reduce aqueous flow through the fistula.
Subsequent treatments if these measures are ineffective include temporary tamponade of the
conjunctiva with a pressure pad, bandage contact lens or collagen shield. Definitive treatment is
with AC re-formation and re-suturing of scleral and/or conjunctival flap.
All of the following are found on examination of Possner-Schlossman syndrome EXCEPT:
fine keratitic precipitates
anterior chamber cells
occludable irido-corneal angles
corneal oedema
Your answer was CORRECT
Explanation
Possner-Schlossman syndrome (PSS) is characterized by an open irido-corneal angle. This
differentiates the acute spike in intra-ocular pressure of PSS from angle-closure glaucoma and/or
inflammatory glaucoma.
A 35-year-old Chinese male presents with a history of blurring of vision at night, with haloes
around lights and associated headache. Episodes last 2-3 hours. On examination, his eyes are quiet
but angles are occludable with a shallow anterior chamber. Optic discs are healthy and visual
fields are full.
What is the most accurate diagnosis in this case?
acute congestive angle closure
primary angle closure glaucoma
sub-acute angle closure
post-congestive angle closure
Your answer was INCORRECT
Explanation
Intermittent or sub-acute angle closure involves intermittent, recurrent episodes of pupillary block,
which are spontaneously relieved. During an attack of sub-acute angle closure, vision is blurred
with haloes around lights due to transient cornea oedema. There may be associated ocular
discomfort or headache, but the eye is usually white. Attacks are recurrent and are usually broken
after 1-2 hours by physiological miosis (e.g. bright light or sleep). Treatment is with prophylactic
peripheral laser iridotomy to prevent acute glaucoma.
A 14-year-old boy with hearing impairment is referred by his renal physician with deteriorating
vision in both eyes. On examination, there are band-like defects and multiple vescicles in the deep
corneal layers. In the anterior chamber, corectopia, ectropion uveae and iris atrophy are seen. The
IOPs are 39 mmHg and there is disc cupping.
What is the most likely diagnosis?
Schwartz-Mazuo syndrome
iridocorneal endothelial syndrome
Fuch's heterochromic iridocyclitis
Alport syndrome
Lowe syndrome
Your answer was CORRECT
Explanation
Figure: slit lamp of posterior polymorphous corneal dystrophy
The clinical description in the question above is consistent with Alport's syndrome. The anterior
chamber findings are the result of posterior polymorphous dystrophy, which is associated with
Alport's. The features of posterior polymorphous dystrophy can mimic ICE syndrome, but the
latter condition is unilateral.
Alport's syndrome is characterised by:
X-linked recessive (80%) and X-linked dominant (5%) inheritance patterns Reference:
NIH National Library of Medicine.
defect in Type IV basement membrane collagen
renal failure secondary to glomerular dysfunction
megalocornea
posterior polymorphous dystrophy
anterior lenticonus
anterior sub-capsular cataract
corneal arcus
peripheral retinal flecks
sensorineural deafness
What proportion of patients with open-angle glaucoma enjoy a substantial decrease in intraocular
pressure during the first year following argon laser trabeculoplasty (ALT)?
20%
50%
40%
80%
Your answer was CORRECT
Explanation
Studies suggest approximately 80% of patients experience a significant decrease in IOP 1 year
after ALT. The procedure generally reduces IOP by 20% to 25%, though this effect tends to
reduce over time.
All of the following are true of diurnal intra-ocular pressure fluctuation EXCEPT:
normal eyes manifest a mean diurnal variation of 5mmHg
IOP is typically highest at night
glaucomatous eyes typically exhibit wider variation
in normal tension glaucoma fluctuations are the same as normal eyes
Your answer was INCORRECT
Explanation
IOP is typically highest in the morning, with 80% of patients peaking between 8am and 12pm.
Other options are true.
Which statement about Nd:YAG laser iridotomy is TRUE?
Nd:YAG is generally more effective for heavily pigmented irides when used alone without argon
laser pre-treatment
the most frequent visually significant complication of Nd:YAG peripheral iridotomy is cystoid
macular oedema
the incidence of spontaneous closure of a previously patent iridotomy is lower with Nd:YAG
iridotomy than with argon iridotomy
unlike argon iridotomies, Nd:YAG laser iridotomy rarely induces elevation in IOP that might
threaten vision
Your answer was INCORRECT
Explanation
Argon iridotomies are more likely to close spontaneously than Nd:YAG iridotomies. Argon laser is often
useful for pre-treating dark irides before Nd:YAG iridotomy because Nd:YAG iridotomy on heavily
pigmented irides is difficult. The most common visually significant complication of both Nd:YAG and argon
laser iridotomies is acute glaucoma.
Iridotomies created with an argon laser (versus Nd:YAG):
require more applications
greater energy
more extensive early oedema and tissue destruction at the margins
more iritis
more pupillary distortion
higher incidence of late closure of the iridotomy
Clinically, the Nd:YAG laser has the disadvantage of more frequent bleeding.
Side-effects of pilocarpine include all of the following EXCEPT:
lacrimation
punctal stenosis
retinal detachment
exacerbation of pupillary block
induced hyperopia
Your answer was CORRECT
Explanation
Ocular side effects of pilocarpine include:
conjunctival vascular congestion
miosis
induced myopia
headaches
retinal detachment
punctal stenosis
Note: while pilocarpine is a treatment for pupil block it can also exacerbate pupil block in some scenarios,
since it increases the convexity of the lens and iris by its effect on the ciliary body and zonules.
The following measures can be taken to limit postoperative hypotony with a Baerveldt implant
EXCEPT:
ligature occlusion of tube
collagen plugs
two-stage procedure
pressure-sensitive valve
Your answer was INCORRECT
Explanation
The Baerveldt implant does not have a pressure-sensitive valve, so hypotony in the early
postoperative period has to be managed by other means.
A two-stage procedure has been recommended to limit early hypotony. During the first operation,
the scleral plate is sutured to the globe without connecting the tube into the anterior chamber. This
procedure is followed by a second operation 2 to 8 weeks later, during which the tube is inserted
into the anterior chamber.
An alternative to limit overfiltration after one-stage installation involves the temporary occlusion
of the tube lumen with a ligature, or with semipermeable, biodegradable collagen lacrimal plugs.
Topical beta blockers reduce IOP primarily by:
increasing aqueous outflow via the trabecular meshwork
reducing aqueous secretion
physiologically deepening the irido-corneal drainage angle
increasing aqueous outflow via the uveo-scleral route
Your answer was CORRECT
Explanation
Beta-blockers reduce intra-ocular pressure by decreasing aqueous secretion. This occurs by
inhibition of beta-2 receptors on the ciliary epithelium.
All are true of pigment dispersion syndrome (PDS) EXCEPT:
men with PDS are more likely to develop glaucoma than women
blacks are less likely to be affected than whites
diagnosis is usually in the sixth to seventh decades
myopes with PDS are more likely to develop glaucoma than non-myopes
Your answer was CORRECT
Explanation
Diagnosis of PDS is usually in the third or fourth decades. Other options are true.
What percentage of aqueous drainage occurs via the conventional route?
75%
90%
10%
25%
Your answer was CORRECT
Explanation
Approximately 90% of aqueous drainage occurs via the trabecular meshwork into the Canal of
Schlemm, where it is drained by the episcleral veins. The uveoscleral or unconventional route
accounts for the remaining 10% of aqueous drainage. In the uveoscleral route, aqueous passes
across the ciliary body into the suprachoroidal space where it is drained by the venous circulation
in the ciliary body, choroid, and sclera.
Reference: American Academy Of Ophthalmology Basic and Clinical Science Course 2017.
"Uveoscleral outflow has been estimated to account for 5%–15% of total aqueous outflow."
Which of the following glaucoma medications is contraindicated in a patient with a history of
paroxysmal tachycardia?
acetazolamide
dipivefrin
timolol
pilocarpine
Your answer was CORRECT
Explanation
Although their use has been in decline, both epinephrine and dipivefrin are beta-2 adrenergic
agonists with potentially serious systemic side effects. They are contraindicated in patients with
heart rhythm abnormalities. Timolol, as a beta antagonist, is not a cardiostimulant (quite the
opposite); neither is pilocarpine.
A patient presents for review 2 weeks following trabeculectomy for primary open angle glaucoma.
On examination, the anterior chamber is deep, the bleb is vascularized, and the IOP is 25mmHg.
Which of the following measures is LEAST likely to be effective in lowering the IOP?
release of sutures
laser iridotomy
ocular compression
needling with sub-conjunctival 5-FU injection
Your answer was CORRECT
Explanation
Failure of filtration can be due to subconjunctival and episcleral fibrosis, bleb encapsulation, over-tight
scleral sutures, scarring of the scleral bleb, blockage of the sclerostomy by blood, uveal tissue or vitreous. It
can be treated (depending on its cause) by:
ocular compression
suture manipulation
bleb needlng
subconjunctival 5-FU injection
bleb revision
Which of the following glaucomatous visual field patterns will most quickly progress to loss of
fixation?
a large superior nasal step encroaching within 10 degrees of fixation
superior and inferior paracentral scotomas with no other defects
superior and inferior nasal steps encroaching to 20 degrees
a central 5-degree island
split fixation to the I4e isopter
Your answer was INCORRECT
Explanation
Split fixation is the presence of visual field loss that comes close to fixation. A typical pattern of
progression is: (i) loss near fixation (paracentral scotoma) to (ii) split fixation to (iii) loss of
fixation. Thus, the eye at greatest risk is not one with a 5-degree central field, but one with the
split-fixation in the horizontal meridian.
Which organism is LEAST likely to be the cause of blebitis following trabeculectomy?
Haemophilus
E. coli
Staphylococcus
Streptococcus
Your answer was CORRECT
Explanation
The most common organisms causing blebitis are Staphylococcus spp., Streptococcus spp and
Haemophilus influenza. Gram negative bacilli such as [Link] are less likely culprits.
A hyperpigmented meshwork is associated with all of the following EXCEPT:
neoplasm
Fuchs' heterochromic iridocyclitis
inflammation
previous trauma
pseudoexfoliation
Your answer was INCORRECT
Explanation
Patients with Fuchs' heterochromic iridocyclitis typically have abnormal vessels present in the angle without
peripheral anterior syndrome (PAS), but they do not have a hyperpigmented meshwork.
Hyperpigmentation should suggest:
pseudoexfoliation
pigment dispersion
previous inflammation, trauma or surgery
uveal melanoma
Which statement is most accurate regarding pigment dispersion syndrome?
it is associated with hypermetropia
the rate of pigment deposition decreases in old age
it is less likely to respond to argon laser trabeculoplasty than primary open angle glaucoma
it is more common in females
Your answer was CORRECT
Explanation
Pigment dispersion is more common in males and is associated with myopia. The rate of pigment
deposition often reduces after the age of 40, perhaps because of the enlarging lens and the
changing shape of the iris-lens diaphragm preventing chaffing of the iris pigment against the
zonules. Pigment dispersion syndrome responds better to ALT than standard POAG because of the
increased pigment in the trabecular meshwork allowing better absorption of laser.
This question came in the FRCS (Glasgow) Part 2 in October 2014.
A 36-year-old man who is a high myope presents with blurred vision and flashing lights. On
examination, the cornea is oedematous, IOP is 46mmHg and gonioscopy shows a wide-open
iridocorneal angle. Dilated funduscopy reveals multiple retinal tears superiorly and a retinal
detachment.
What is the most likely diagnosis?
Schwartz-Matsuo syndrome
plateau iris syndrome
aqueous misdirection syndrome
Posner-Schlossman syndrome
pigment dispersion syndrome
Your answer was CORRECT
Explanation
Schwartz-Matsuo syndrome occurs when photoreceptor outer segments released from retinal tears
and detachments migrate into the anterior chamber and block the trabecular meshwork causing the
IOP to rise. The condition usually resolves once the detachment is repaired.
Which type of glaucomatous nerve damage is most common in females with vasopasm or
migraine?
concentrically enlarging
focal ischaemic
senile sclerotic
mixed
Your answer was CORRECT
Explanation
The sub-types of glaucomatous optic nerve damage are:
Type 1 (focal ischaemic): characterized by a focal notch in the neuroretinal rim. It usually occurs in
females with a history of migraine or vasospasm. It is typically associated with a localised field
defect near to fixation.
Type 2 (myopic glaucomatous): associated with a temporal crescent in the absence of degenerative
myopia. It is associated with dense superior or inferior field loss and tends to occur in younger male
patients.
Type 3 (senile sclerotic): characterised by a shallow saucerised cup and a gently sloping rim. As its
name suggests, it typically occurs in elderly patients and is associated with hypertension and
cardiovascular disease.
Type 4 (concentrically enlarging): characterised by thinning of the entire rim in the absence of
notching. It is associated with a high IOP at presentation and tends to occur in younger paints. Field
loss is frequently diffuse.
Type 5: mixed
Which topical IOP-lowering agent is most likely to cause a bitter taste?
timolol
bimatoprost
dorzolamide
apraclonidine
Your answer was INCORRECT
Explanation
The carbonic-anhydrase inhibitors dorzolamide (trusopt) and brinzolamide (azopt) commonly
cause a transient bitter taste.
Which of the following is generally NOT a cause of increased episcleral venous pressure?
Sturge-Weber syndrome
orbital varix
thyroid ophthalmopathy
superior vena cava syndrome
contralateral carotid artery stenosis
Your answer was CORRECT
Explanation
Contralateral carotid artery stenosis is not a cause of raised episcleral venous pressure.
What are you most likely to find on gonioscopy in iridocorneal endothelial syndrome:
angle recession
peripheral anterior synechae
peripheral neovascularization
hyperpigmentation
Your answer was CORRECT
Explanation
ICE syndrome is caused by an abnormal corneal endothelial cell layer which proliferates and
migrates across the angle and onto the iris. Glaucoma occurs in about 50% of cases, due to
synechial angle closure secondary to contraction of this abnormal tissue. PCR suggests a possible
herpes simplex viral origin to the disease process.
A patient presents 5-days after routine trabeculectomy surgery with deteriorating vision. Vision in
the operated eye is hand motions, IOP is 40 mmHg and the anterior chamber is flat. Siedel test is
negative. A B-scan shows kissing choroidal detachments.
All of the following are risk factors associated with the underlying condition EXCEPT?
aphakia
previous vitrectomy
hyperopia
pre-operative elevated IOP
Your answer was CORRECT
Explanation
The findings are suggestive of a delayed suprachoroidal haemorrhage.
Risk factors associated with this condition include:
aphakia
pseudophakia
myopia
previous vitrectomy
preoperative IOP greater than 30 mmHg
Where is the standard reference plane taken in Heidelberg retinal topography (HRT) of the optic
disc?
on the surface of the optic nerve head boundary line at the nasal disc edge along the horizontal
midline
on the surface of the optic nerve head boundary line at the temporal disc edge along the horizontal
midline
50 um below the optic nerve head boundary line at the nasal disc edge along the horizontal
midline
50 um below the optic nerve head boundary line at the temporal disc edge along the
horizontal midline
Your answer was INCORRECT
Explanation
This location is taken as the standard default reference plane because it corresponds to the centre
of the papillomacular bundle of the RNFL. This papillomacular bundle is usually preserved until
late in glaucoma, so it would be expected to have a stable thickness over time.
Which of the following miotics is a direct-acting parasympathomimetic agent?
echothiophate
pyridostigmine
carbachol
demecarium
Your answer was CORRECT
Explanation
Direct-acting miotics interact directly with the acetylcholine receptor (e.g. pilocarpine and
carbachol), whereas indirect-acting agents increase the activity of native acetylcholine at the
synaptic junction by blocking its enzymatic degradation (e.g. echothiophate, demecarium and
pyridostigmine).
A 65-year-old phakic, Caucasian man undergoes trabeculectomy for POAG with adjunctive
mitomycin C. On post-operative day 1, the IOP is 2 mmHg, the bleb is flat, anterior chamber
shallow, Seidl's is negative and he has large choroidal elevations.
What is the most likely cause of this clinical picture?
aqueous misdirection
sub-clinical bleb leak or sweating
ciliary body shutdown
overdrainage
patient eye rubbing
Your answer was CORRECT
Explanation
The description is most in keeping with ciliary body shutdown. A bleb leak would produce similar
signs except for a positive Siedl's test. Over-drainage (including by eye-rubbing) would produce a
large bleb with low pressure while aqueous misdirection produces a flat bleb and shallow AC with
a raised (not lowered) IOP.
A patient presents for his 1-day post-trabeculectomy review. On examination, the anterior
chamber is very shallow with peripheral irido-corneal touch. The IOP is 50mm. The iridotomy is
patent. The bleb is flat and Seidel test is negative.
What is the most likely diagnosis:
pupillary block
scleral flap leakage
bleb leakage
aqueous misdirection syndrome
Your answer was CORRECT
Explanation
A shallow anterior chamber with high IOP post-trabeculectomy can be the result of:
Pupillary block: this is due to a non-patent peripheral iridotomy with iris bombe. The bleb is flat and
Siedl's is negative
Aqueous misdirection: this is due to aqueous being directed posteriorly into the vitreous. The AC is
shallow, the IOP high, the bleb flat and Seidel negative. However, the iridotomy in this case is
patent with no iris bombe.
A shallow anterior chamber with low IOP post-trabeculectomy can be the result of:
Scleral flap leak: the bleb is well formed and Seidel test negative
Conjunctival bleb leak: the bleb is flat and Seidel test positive
Laser trabeculoplasty is most likely to be effective in which of the following types of uncontrolled
glaucoma?
Iridocorneal endothelial syndrome
Angle recession glaucoma
Inflammatory glaucoma
Pigmentary glaucoma
Your answer was CORRECT
Explanation
Laser trabeculoplasty effectively lowers IOP in patients with:
POAG
pigmentary glaucoma
pseudoexfoliation
It is ineffective and may actually worsen the IOP in eyes with:
inflammatory glaucoma
recessed angles
membranes in the angle
young patients with developmental defects
What is the most distinguishing sign of Chandler syndrome setting it apart from other ICE
syndromes:
corectopia
'hammered-silver' corneal endothelium
diffuse iris naevus
pseudopolycoria
Your answer was CORRECT
Explanation
The ICE syndrome consists of the following three overlapping disorders:
Progressive iris atrophy: severe iris changes including pseudopolycoria, corectopia, iris atrophy.
Cogan-Reese syndrome: diffuse iris naevus or iris nodules. Iris changes also prominent in 50% of
cases.
Chandler syndrome: hammered-silver corneal endothelial changes. Iris changes present in 40%.
Glaucoma less prominent than the other 2 forms.
The anterior chamber angle structures cannot be directly visualised because:
opaque sclera overhangs the anterior chamber angle
there is total internal reflection at the cornea-air interface
there is excessive scattering of light reflected from the angle off the iris
there is total internal reflection at the aqueous-corneal endothelial interface
Your answer was CORRECT
Explanation
The anterior chamber angle cannot be viewed directly through the cornea because there is total
internal reflection of light at the cornea-air interface. Gonioscopy replaces this interface with a
new cornea-lens interface. The difference in refractive indices is reduced, so total internal
reflection does not occur.
Which is FALSE regarding plateau iris syndrome?
acute angle closure occurs due to bunching up of the iris in the angle
the ciliary processes are anteriorly positioned
treatment with pilocarpine and peripheral iridotomy is usually effective
the iris plane is flat
Your answer was INCORRECT
Explanation
In plateau iris configuration (PIC) the iris plane is flat as opposed to typical pupil-block acute
congestive glaucoma, where it is convex. Also, the anterior chamber is normal in depth centrally
rather than shallow. Otherwise, the signs can be similar in plateau iris configuration as for acute
congestive glaucoma with occluded irido-corneal angles, at times severely raised IOP, corneal
oedema etc. Pilocarpine combined later with a peripheral iridotomy is often effective for plateau
iris configuration (PIC). However, plateau iris syndrome (PIS) is a post-operative diagnosis, given
to those patients in whom peripheral iridotomy has not been effective at relieving the angle
closure. Peripheral iridoplasty can be used to reduce the iris hump peripherally. [Reference]
Which is FALSE regarding the effect of steroids on intra-ocular pressure?
systemic steroids are more prone to cause elevation of IOP than topical steroids
patients with POAG are more likely to be steroid responders than the general population
steroid use increases the production of myocilin by trabecular meshwork
fluorometholone raises IOP less than betamethasone
Your answer was CORRECT
Explanation
Systemic steroids are less prone to cause elevation of IOP than topical steroids, but they are more
likely to induce cataract. Other options supplied are true.
All of the following statements about infantile glaucoma are true EXCEPT:
a common gonioscopy finding is an abnormally anterior iris insertion
one third of cases are bilateral
it is more common in males
approximately 50% of cases are primary with no ocular or systemic associations
Your answer was INCORRECT
Explanation
The majority (about two-thirds) of cases are bilateral. Other statements above are true.
The most important growth factor in the pathogenesis of neovascular glaucoma is:
erythropoietin
fibroblast growth factor
vascular endothelial growth factor
insulin-like growth factor
transforming growth factor alpha
Your answer was CORRECT
Explanation
The most important growth factor in the pathogenesis of neovascular glaucoma is vascular
endothelial growth factor (VEGF).
A 73-year old lady presents with left-sided headache and blurred vision. On examination, she has
left ciliary flush, left corneal oedema, fixed mid-dilated left pupil and bilateral shallow anterior
chambers. Intra-ocular pressures are 12mm Hg right and 73 mm Hg left.
What is the most important immediate treatment?
oral glycerine
topical pilocarpine 2%
intravenous acetazolamide
topical dexamethasone
oral acetazolamide
Your answer was CORRECT
Explanation
This patient has acute angle closure glaucoma. The most important initial treatment is stat iv
acetazolamide 500mg to reduce the intra-ocular pressure. Oral glycerine and oral acetazolamide
may be administered as adjuncts if the IOP has not responded satisfactorily to stat iv
acetazolamide. Topical pilocarpine will not be effective at such a high IOP due to iris ischaemia
though it is prudent to administer once the IOP is controlled, and also to the fellow eye. Topical
dexamethasone is indicated to control the inflammatory component of PACG but it is not the
immediate priority.
A patient is taking the Humphrey automated perimetery test. The machine projects a light at his
blind spot and the patient presses the button. What does this response MOST likely represent?
fixation loss
false-negative response
short-term fluctuation
false-positive response
Your answer was CORRECT
Explanation
The Humphrey automated perimeter has a number of ways to test the reliability of the test taker.
A fixation loss occurs when the patient responds as if seeing a light when a target is displayed in
his blind spot.
A false-negative response occurs when the patient fails to respond to a supra-threshold stimulus at
a location that would be expected to be seen. This response may indicate a patient who is falling
asleep or losing interest.
Intermittently, the perimeter will pause and the motorized light will change position making a
familiar noise, but no stimulus will be presented. If the patient presses the button, a false-positive
response is recorded. A nervous or trigger-happy patient may have a high false-positive rate.
Which is FALSE regarding Peter's anomaly?
the corneal opacification is typically progressive until the age of 5 years
the condition can be associated with both lens and iris adhesions to the corneal endothelium
the condition results from a developmental problem with neural crest cells
the peripheral cornea is unaffected
Your answer was INCORRECT
Explanation
Peter's anomaly results from faulty migration of neural crest cells. Neural crest cells usually
migrate between the surface ectoderm of the cornea and the separating lens. As a result of faulty
separation, both lens and iris may remain adherent to the central cornea, causing a central corneal
opacity. Both Descemet's membrane and layers of the posterior stroma may be absent. The
peripheral cornea is characteristically clear because Descemet's membrane and the endothelium
are intact. In many cases, the corneal opacity will decrease over time.
The rate of glaucoma in patients with Sturge-Weber syndrome is:
10%
1%
5%
30%
Your answer was CORRECT
Explanation
30% of Sturge-Weber syndrome patients develop glaucoma. Most (60%) occur in patients under
the age of 2 years. Isolated trabeculodysgenesis is important in the pathophysiology of early-onset
glaucoma. Meanwhile, raised episcleral venous pressure is important in late-onset glaucoma.
All of the following medications may be associated with the induction or aggravation of angle-
closure glaucoma EXCEPT?
cyclopentolate
pilocarpine
thiazide diuretics
oral antihistamines
Your answer was INCORRECT
Explanation
Both mydriatics and miotics can precipitate angle-closure in eyes with shallow anterior chambers.
This is true for both topical medications and systemic drugs that affect the pupil. Examples
include antihistamines, which can have anticholinergic activity.
Which is FALSE regarding the risk of developing glaucoma:
a nocturnal dip in blood pressure may reduce the risk of glaucoma
diabetes increases the risk
a rhegmatogenous retinal detachment increases the risk
blacks are at greater risk than whites
Your answer was CORRECT
Explanation
A reduction in perfusion pressure of the optic nerve (for example by a nocturnal dip in blood pressure)
increases the risk of glaucoma. All other options are true.
Three days post-trabeculectomy a patient has a shallow anterior chamber with an IOP of 5mmHg.
The bleb is flat and Seidel test is negative. Fundoscopy is normal.
What is the diagnosis?
overfiltration
pupillary block
aqueous misdirection
bleb leak
ciliary body shutdown
Your answer was CORRECT
Explanation
A shallow anterior chamber with high IOP post-trabeculectomy can be the result of:
Pupillary block: this is due to a non-patent peripheral iridotomy with iris bombe. The bleb
is flat and Siedl's is negative
Aqueous misdirection: this is due to aqueous being directed posteriorly into the vitreous.
The AC is shallow, the IOP high, the bleb flat and Seidel negative. However, the
iridotomy in this case is patent with no iris bombe.
A shallow anterior chamber with low IOP post-trabeculectomy can be the result of:
Scleral flap leak: the bleb is well formed and Seidel test negative
Conjunctival bleb leak: the bleb is flat and Seidel test positive
Ciliary body shutdown: the bleb is flat and Seidel test is negative
What percentage of patients with pigment dispersion develop raised intra-ocular pressure or
glaucoma after 15 years:
80%
2%
10%
33%
Your answer was INCORRECT
Explanation
Approximately one-third of patients with pigment dispersion ultimately develop raised intra-ocular pressure
and/or glaucoma after 15 years.
What percentage of patients with pigment dispersion develop raised intra-ocular pressure or
glaucoma after 15 years:
80%
2%
10%
33%
Your answer was INCORRECT
Explanation
Approximately one-third of patients with pigment dispersion ultimately develop raised intra-ocular pressure
and/or glaucoma after 15 years.
What percentage of optic nerve axons may be lost before visual field changes are detected by
Goldmann perimetry?
10%
25%
15%
50%
Your answer was INCORRECT
Explanation
Pathological changes of the optic nerve precede visual field changes. In fact, up to 50% of optic nerve axons
can be lost before any change is detected on the Goldmann visual field. This observation has stimulated
interest in measuring the retinal nerve fiber layer thickness via computer using technologies such as the
OCT. These technologies will likely be useful but will never replace the need for a careful ophthalmoscopic
exam and clinical assessment.
Sampaolesi's line describes:
pigmentation anterior to Schwalbe's line
pigmentation along Schwalbe's line
pigmentation posterior to Schwalbe's line
pigmentation of the scleral spur
Your answer was CORRECT
Explanation
Sampaolesi's line describes pigmentation anterior to Schwalbe’s line and occurs in pigment dispersion
syndrome and pseudoexfoliation.
Schwartz's syndrome is caused by:
deformed red blood cells from the vitreous cavity blocking the trabecular meshwork
forward rotation of the lens-iris diaphragm
photoreceptor outer segments blocking the trabecular meshwork
ciliary body and choroidal oedema
haemoglobin-laden macrophages blocking the trabecular meshwork
Your answer was CORRECT
Explanation
Schwartz's syndrome is high IOP associated with a rhegmatogenous retinal detachment. Photoreceptor outer
segments (+- RPE pigment) migrate transvitreally into the aqueous, block the trabecular outflow pathways,
and result in IOP elevation.
A 75-year-old lady with a known previous right central retinal vein occlusion presents with a 1-
week history of painful right eye with associated headache. On examination, vision is hand
movements, there is right-sided corneal oedema, extensive fibrovascular rubeosis with an
occluded angle 360 degrees, a dense white cataract precluding fundal views and an IOP of
55mmHg. She is currently on no regular medication.
What would be the most appropriate definitive treatment:
argon PRP
retro-bulbar alcohol
trans-scleral diode cycloablation
topical beta blockade
iv acetazolamide
Your answer was CORRECT
Explanation
This lady requires trans-scleral cycloablation to reduce the intra-ocular pressure. She may also
require topical steroid and atropine to reduce pain and inflammation, with consideration given to
supplementary IOP lowering therapy in the form of topical beta blockade and/or systemic
acetazolamide. However, the definitive treatment is cycloablation.
Consideration should also be given in this case to retinal ablation to regress the new vessels, if this
has not been done previously. If it were to be performed on this visit, it could not be Argon PRP
because there are no clear fundal views - it would have to be trans-scleral retinal diode.
Retro-bulbar alcohol injection is a treatment of last resort and should not be considered at this
juncture (first presentation with rubeotic glaucoma).
Which method of gonioscopy is considered best for evaluating a patient with potential traumatic
(angle-recession) glaucoma?
Goldmann
Zeiss
Koeppe
Sussman
Your answer was INCORRECT
Explanation
Koeppe gonioscopy is considered best for evaluating a patient with potential angle recession
because this system allows easier comparison of one eye with the fellow eye, or one portion of the
angle with another.
Note that the Koeppe, Richardson, Barkan, Wurst and Swan-Jacob lenses are used in direct
gonioscopy, while the Goldmann, Zeiss, Posner and Sussman lenses are used in indirect
gonioscopy.
All are true of 5-FU utilization in trabeculectomy EXCEPT:
it is the anti-metabolite of choice in young patients
it inhibits fibroblastic proliferation
it increases the risk of late bleb leakage
it is toxic to the corneal epithelium
Your answer was INCORRECT
Explanation
5-FU inhibits DNA synthesis and thereby inhibits fibroblast proliferation. It is the anti-metabolite
of choice in the elderly undergoing high-risk trabeculectomy, because it is less potent than
mitomycin C. Like all anti-metabolites, it increases the risk of over-filtration, hypotony and late
bleb leakage. It is toxic to the corneal epithelium, which should be rinsed thoroughly after use.
Which glaucoma agent is relatively contraindicated in patients with a history of depression or
other psychiatric illness?
latanoprost
epinephrine
dipivefrin
pilocarpine
timolol
Your answer was INCORRECT
Explanation
Beta-antagonists, in general, can cause fatigue, dizziness, and depression at therapeutic doses and
should be used with caution in patients with a history of severe depression.
Which secondary glaucoma is caused by elevated episcleral venous pressure:
pseudoexfoliation glaucoma
phacolytic glaucoma
Sturge-Weber syndrome glaucoma
uveitic glaucoma
ghost cell glaucoma
Your answer was CORRECT
Explanation
Secondary glaucoma can be caused by impaired aqueous outflow as a result of elevated episcleral venous
pressure. This occurs in:
carotid-cavernous fistula
Sturge-Weber syndrome
obstruction of the superior vena cava.
The other options supplied above (ghost cell glaucoma, phacolytic glaucoma, uveitic glaucoma,
pseudoexfoliation glaucoma) are secondary glaucomas caused by physical obstruction or clogging of the
meshwork itself.
All are factors in the pathogenesis of inflammatory glaucoma EXCEPT:
trabeculitis
increased aqueous production
inflammatory cell debris
peripheral anterior synechae
iris bombe
Your answer was INCORRECT
Explanation
Inflammatory or uveitis-related raised IOP is multifactorial. It may be due to:
Seclusio pupillae: that is, 360 degree posterior synechae resulting in iris bombe and closed-angle
glaucoma
Peripheral anterior synechae: obstructing the meshwork with a closed angle
Inflammatory debris: clogging the meshwork but with an open angle
Trabeculitis: affecting the drainage function by direct inflammation of the meshwork
However, increased aqueous production is NOT a factor in the pathogenesis of inflammatory glaucoma.
Rather, aqueous production is decreased during inflammatory episodes, which can often mask underlying
glaucoma, which is only evident between episodes of active inflammation when ciliary function returns to
normal
Which one of the following procedures has the highest incidence of hypotony?
Trabeculectomy with mitomycin C
Trabeculectomy with 5-FU
Baerveldt glaucoma tube
Full-thickness sclerectomy
Your answer was INCORRECT
Explanation
The incidence of hypotony is highest with full-thickness procedures such as a posterior lip
sclerectomy. Because of this, these procedures are performed less commonly today. Partial
thickness procedures, including trabeculectomies with antimetabolites, have lower rates of
hypotony. Glaucoma drainage devices (such as the Baerveldt or Molteno implant) are intermediate
in incidence depending on the type of implant and whether a ligature or other device is used to
occlude the drainage tube.
Which best describes the mechanism of action of latanoprost:
reduces aqueous secretion
increases aqueous outflow via the trabecular meshwork
physiologically deepens the irido-corneal drainage angle
increases aqueous outflow via the uveo-scleral route
Your answer was CORRECT
Explanation
Prostaglandin analogues such as latanaprost and travaprost work by increasing aqueous outflow,
primarily via the uveo-scleral outflow route. Bimatoprost in addition to increasing uveo-scleral
outflow, is thought to enhance trabecular outflow.
Which anaesthetic is recommended for the examination of suspected congenital glaucoma?
intravenous propofol
intravenous ketamine
intravenous thiopental
halothane gas
Your answer was CORRECT
Explanation
Evaluation of congenital glaucoma should be performed under general anaesthesisa with
intravenous ketamine, as other anaesthetic agents may lower IOP causing an under-estimation of
genuine IOP.
Use of ketamine among anaesthetists is declining generally and if your anaesthetist is not
comfortable with its use, then IOP should be measured as soon after induction as possible to
minimise IOP lowering effects of anaesthesia
A lady with POAG has a 0.7 vertical cup-disc ratio with superior thinning of the neuroretinal rim,
an arcuate scotoma and a visual field MD of -8dB.
What grade of glaucomatous damage does she have?
Grade 4 (end-stage)
Grade 3 (severe)
Grade 2 (moderate)
Grade 1 (mild)
Your answer was INCORRECT
Explanation
Grading of glaucomatous damage:
Mild (Grade 1): minimal cupping, a nasal step or paracentral scotoma and a MD < -6dB
Moderate (Grade 2): thinning of neuroretinal rim, an arcuate scotoma and a MD < -12dB
Severe (Grade 3): marked cupping, extensive VF loss including the central 5 degrees and MD > -
12dB
End-Stage (Grade 4): gross disc cupping and a small residual field
A 58-year-old man presents with a 2-day history of a red, painful right eye with reduced vision.
He had a trabeculectomy for primary open angle glaucoma in this same eye 6 months ago. On
examination, vision is 6/60, there is an inflamed, milky bleb superiorly, a hypopyon and no fundal
views.
What would a Gram stain of the vitreous fluid of this patient most likely show?
White cells with no organisms
Gram positive cocci in chains
Branching pseudohyphal forms
Gram-negative coccobacilli
Gram-positive rods
Your answer was INCORRECT
Explanation
This patient has bleb-associated endophthalmitis.
The most common organisms responsible in order of frequency are:
Streptococcus species (Gram-positive cocci in pairs or chains)
Haemophilus influenzae (Gram-negative coccobacilli)
Staphylococcus epidermidis (Gram-positive cocci in clusters)
Gram-negative species
The advantages of selective laser trabeculoplasty (SLT) over ALT in treating glaucoma include all
of the following EXCEPT:
general structure of trabecular meshwork more intact post-SLT
SLT is more effective at lowering IOP
SLT selectively targets pigmented trabecular meshwork cells
SLT uses a potentially repeatable laser
Your answer was CORRECT
Explanation
SLT targets only the melanin-containing cells in the trabecular meshwork, without causing
thermal damage to adjacent non-pigmented trabecular meshwork cells and underlying trabecular
beams. This is why, unlike ALT, SLT is repeatable several times. ALT patients can receive two
treatments in a lifetime, whereas SLT patients can receive two treatments a year. SLT has not been
proven to be more effective than ALT.
What is the most common cause of bleb failure after trabeculectomy?
late bleb leak
episcleral fibrosis
closure of the internal sclerostomy
bleb encapsulation (Tenon's cyst formation)
Your answer was INCORRECT
Explanation
The most frequent cause of failure after filtration surgery is bleb scarring due to episcleral fibrosis.
This excessive healing response is largely due to the proliferation of fibroblasts and the production
of collagen and glycosaminoglycans. The antimetabolites 5-FU and MMC have been used to
modulate wound healing after filtration surgery
Regarding ICE syndrome, which is FALSE?
HSV has been proposed in the aetiology
trabeculectomy with mytomycin C will usually control the IOP longterm
broad peripheral anterior synechae are common on gonioscopy
in Chandler syndrome there are endothelial abnormalities
Your answer was INCORRECT
Explanation
Glaucoma in ICE syndrome is due to synechial angle closure secondary to contracture of
abnormal, proliferative corneal endothelial tissue. Glaucoma is often very difficult to treat and
trabeculectomy with anti-metabolites is frequently unsuccessful due to late-onset bleb failure.
Glaucoma drainage devices are eventually required in many cases.
This question appeared in the 2014 FRCOphth Part 2.
Day 2 post-trabeculectomy a patient returns for routine review. The anterior chamber is shallow
and IOP is 34 mmHg. The bleb is flat and Seidel test is negative. The peripheral iridotomy is
patent and fundoscopy is unremarkable.
What is the diagnosis?
sclerostomy block
aqueous misdirection
pupillary block
suprachoroidal haemorrhage
Your answer was INCORRECT
Explanation
The findings are in keeping with aqueous misdirection syndrome.
A shallow anterior chamber with high IOP post-trabeculectomy can be the result of:
Pupillary block: this is due to a non-patent peripheral iridotomy with iris bombe. The bleb is flat and
Siedl's is negative
Aqueous misdirection: this is due to aqueous being directed posteriorly into the vitreous. The AC is
shallow, the IOP high, the bleb flat and Seidel negative. However, the iridotomy in this case is
patent with no iris bombe.
A shallow anterior chamber with low IOP post-trabeculectomy can be the result of:
Scleral flap leak: the bleb is well formed and Seidel test negative
Conjunctival bleb leak: the bleb is flat and Seidel test positive
Ciliary body shutdown: the bleb is flat and Seidel test is negative
In which condition is a drainage implant MOST likely to be used as a first-line surgical option?
pseudoexfoliation glaucoma
traumatic angle recession glaucoma
primary congenital glaucoma
normal tension glaucoma
Your answer was CORRECT
Explanation
Drainage implants are considered in the following circumstances:
previous failed trabeculectomy with adjuvant antimetabolite
neovascular glaucoma
traumatic angle recession glaucoma
eyes with severe conjunctival scarring
congenital glaucomas where conventional procedures (e.g goniotomy or trabeculotomy) have failed
All are true of primary congenital glaucoma EXCEPT:
it is more common in boys
it is usually bilateral
Haab's striae represent fresh breaks in Descemet's membrane
buphthalmos develops if IOP is raised before 3 years of age
corneal haze is often the first sign noticed by parents
Your answer was INCORRECT
Explanation
Haab's striae represent healed breaks in Descemet's membrane and appear as horizontal
curvilinear lines. Other options above are true.
Infantile or juvenile glaucoma is LEAST likely to be encountered in which of the following
phakomatoses:
von Hippel Lindau
tuberous sclerosis
neurofibromatosis
Sturge-Weber syndrome
Your answer was CORRECT
Explanation
Among the phakomatoses, unilateral congenital glaucoma is seen in:
Sturge-Weber syndrome (25% of cases, often associated with an angioma on the upper lid)
von-Recklinghausen's syndrome (especially if plexiform neurofibroma of the upper lid)
von Hippel-Lindau disease
Infantile or juvenile glaucoma is LEAST likely to be encountered in which of the following
phakomatoses:
von Hippel Lindau
tuberous sclerosis
neurofibromatosis
Sturge-Weber syndrome
Your answer was CORRECT
Explanation
Among the phakomatoses, unilateral congenital glaucoma is seen in:
Sturge-Weber syndrome (25% of cases, often associated with an angioma on the upper lid)
von-Recklinghausen's syndrome (especially if plexiform neurofibroma of the upper lid)
von Hippel-Lindau disease
In a child with primary congenital glaucoma and a cloudy cornea, which surgical procedure
should be used initially?
trabeculotomy
goniotomy
trabeculectomy with mitomycin C
cyclophotocoagulation
Your answer was CORRECT
Explanation
Management of congenital glaucoma involves surgery, either goniotomy (70% success) or
trabeculotomy (70% to 80% success). A goniotomy requires the cornea to be clear enough to view
the meshwork. In the scenario presented, a trabeculotomy ab externo is the procedure of choice
because of the cloudy cornea. In general, for primary congenital glaucoma, if repeated attempts at
goniotomy and/or trabeculotomy fail, filtering surgery with mitomycin C or a glaucoma drainage
tube are considered. Topical medication, such as beta-blockers, miotics, or carbonic anhydrase
inhibitors can temporise and potentially clear the cornea enough to allow goniotomy to be
performed. Cyclophotocoagulation and other cyclodestructive procedures usually are reserved for
eyes that have undergone other unsuccessful surgeries
A 72-year-old Korean woman presents with a 4-hour history of acute, right eye pain, blurred
vision, and redness of her eye. She has a cloudy cornea with epithelial oedema, shallow anterior
chamber, 4-mm non-reactive pupil, and IOPs of 62 mmHg OD and 17 mmHg OS. Gonioscopy of
the left eye reveals a narrow potentially occludable angle with virtually no angle structures visible.
She has never had any previous similar episodes.
What other finding would be MOST likely on examination:
keratic precipitates
optic nerve pallor and cupping
glaukomflecken
optic nerve hyperaemia and swelling
Your answer was INCORRECT
Explanation
Hydropic degeneration and impaired axoplasmic flow cause swelling and hyperaemia of the optic
nerve in acute angle closure glaucoma. Glaukomflecken and optic nerve pallor and cupping would
indicate previous episodes of angle closure glaucoma, which the patient denies. Cell and flare may
be seen with prolonged attacks, but keratic precipitates are rarely seen.
Which one of the following is the MOST important medication to discontinue as far before
glaucoma surgery as possible?
Dipivefrin
Echothiophate
Pilocarpine
Timolol
Your answer was CORRECT
Explanation
Echothiophate is a strong, relatively irreversible cholinesterase inhibitor. This drug causes a
disruption of the blood-aqueous barrier, which may cause increased inflammation after intraocular
surgery in eyes pre-treated with echothiophate. Indirect agents also block other cholinesterases,
including plasma pseudocholinesterase, which deactivates succinylcholine. Patients may be
paralyzed for extended periods of time after anaesthesia with succinylcholine and need to be
warned about this. For these reasons, it is usually advisable to discontinue the drug several weeks
before surgery.
Which of the following agents used in general anaesthesia tends to lower intraocular pressure?
1. Halothane
2. Ketamine
3. Succinylcholine
1, 2 and 3
2 and 3
1 and 3
2 only
1 only
Your answer was INCORRECT
Explanation
Ketamine and succinylcholine elevate intraocular pressure. Ketamine only does so at high doses.
Most halothane anaesthetic agents lower intraocular pressure.
In which condition is argon laser trabeculoplasty MOST likely to be effective:
chronic angle closure glaucoma
Possner-Schlossman syndrome
normal tension glaucoma
angle recession glaucoma
Your answer was CORRECT
Explanation
ALT is successful in:
Primary open angle glaucoma
pigmentary glaucoma
pseudoexfoliation glaucoma
normal tension glaucoma
It is generally ineffective in:
paediatric glaucomas
secondary glaucomas
All of the following are indirect-acting parasympathomimetic agents EXCEPT?
echothiophate
pilocarpine
pyridostigmine
demecarium
Your answer was CORRECT
Explanation
Direct-acting parasympathomimetics interact directly with the acetylcholine receptor (e.g.
pilocarpine and carbachol), whereas indirect-acting agents increase the activity of native
acetylcholine at the synaptic junction by blocking its enzymatic degradation (e.g. echothiophate,
demecarium and pyridostigmine).
A 54-year-old Nigerian man with a history of open-angle glaucoma presents with a Goldmann
visual field documenting a superior nasal step to the I4e isopter in the right eye. When he returns
with a deteriorated visual field one year later, the most likely form of deterioration is:
a superior paracentral scotoma
an inferior Bjerrum scotoma
encroachment of his superior nasal step toward fixation
an inferior nasal step
Your answer was INCORRECT
Explanation
Areas of retina and/or optic nerve damaged by glaucoma are believed to be more vulnerable to
ongoing damage at lower intraocular pressures. New defects also may appear, of course, but
generally accompany progression of previous defects.
Primary congenital glaucoma is usually:
autosomal dominant
autosomal recessive
sporadic
X-linked
Your answer was CORRECT
Explanation
Most cases of primary congenital glaucoma are sporadic. About 10% are inherited as autosomal
recessive with incomplete penetrance.
This question appeared in the 2014 FRCOphth Part 2.
A 29-year-old man has presented over a 5-year period with several episodes of unilateral elevation
of IOP to the 40 to 50 mmHg range. During these episodes, there is mild ciliary flush, fine keratic
precipitates and faint flare. The anterior chamber is otherwise normal. Each episode seems to
respond well to topical corticosteroids and topical and systemic aqueous suppression.
What is the most likely diagnosis?
Sarcoidosis
Juvenile rheumatoid arthritis
Posner-Schlossman syndrome
Fuchs' heterochromic iridocyclitis
Your answer was CORRECT
Explanation
The clinical picture described is characteristic of Posner-Schlossman syndrome. Fuchs'
heterochromic iridocyclitis can present with a similar picture but with iris hypochromia and gray-
white nodules on the anterior iris.
Focal ischaemic optic nerve damage from glaucoma is typically associated with:
altitudinal defect
inferior arcuate scotoma
enlarged blind spot
localized field defect near to fixation
Your answer was INCORRECT
Explanation
Focal ischaemic glaucoma (subtype 1) is characterized by a focal notch in the neuroretinal rim. It
usually occurs in females with a history of migraine or vasospasm. It is typically associated with a
localised field defect near to fixation.
Which is TRUE regarding beta-blockers:
carteolol has instrinsic sympathomimetic activity
beta-blockers increase aqueous drainage
tachyphylaxis occurs in 90% of patients on beta-blockers
selective beta-1-blockers have less effects on the heart than non-selective beta-blockers
Your answer was CORRECT
Explanation
The heart contains mainly beta-1 receptors, while beta-2 receptors are found in the lungs, GI tract,
and vascular smooth muscle. Selective beta-1 blockers therefore have less effect on the lungs (ie
are cardioselective) e.g. bisoprolol, atenolol and betoxolol. Beta-blockers reduce IOP by
decreasing aqueous production with little effect on aqueous drainage. Of the glaucoma options,
carteolol has intrinsic sympathomimetic activity, so will induce less bradycardia and postural
hypotension than other options. Tachyphylaxis can occur with beta-blockers, but is not as
common as 90%.
What is the cumulative risk of glaucoma at 5 years in eyes with pseudoexfoliation?
5%
1%
25%
60%
Your answer was CORRECT
Explanation
Different studies have reported different rates. However, the cumulative risk of glaucoma in eyes
with PXF is considered about 5% at 5 years and 15%-30% at 10-15 years.
In a patient with a trabeculectomy bleb, which of the following signs may occur in both
endophthalmitis and blebitis:
vitritis
loss of red reflex
hypopyon
a milky white bleb
Your answer was INCORRECT
Explanation
Blebitis is an infection localised to a trabeculectomy bleb, although there is a risk of intraocular
spread to cause endophthalmitis. Discriminating between the two is important for clinical
management. Blebitis may be treated with topical antibiotics with or without steroid cover, while
endophthalmitis requires an intravitreal tap and intravitreal antibiotics. In both endophthalmitis
and blebitis, there is a milky white infected bleb, which is the source of the infection. However,
with endophthalmitis there is associated loss of vision, loss of red reflex, significant AC reaction
with hypopyon and vitritis
All of the following intra-ocular pressure-lowering agents should be used with caution in
pregnancy due to possible teratogenic effects EXCEPT:
beta-blockers
alpha-2 agonists
prostaglandin analogues
carbonic anhydrase inhibitors
Your answer was INCORRECT
Explanation
Prostaglandin analogues, carbonic anhydrase inhibitors and beta-blockers have all been shown in
animal studies to have teratogenic effects (though at levels well above their therapeutic use in
humans). Alpha-2 agonists such as brimonidine have not been shown to cause teratogenic effects -
therefore they may be the safest drug in early pregnancy. However, their use in late pregnancy and
during breast-feeding should be avoided because they can cross the blood-brain barrier causing
CNS depression and apnoea in the neonate
All are side-effects of topical beta-blockers EXCEPT:
bronchospasm
decreased libido
bradycardia
increased aqueous tear secretion
Your answer was CORRECT
Explanation
Side-effects of beta-blockers include:
puntate corneal erosions
local allergy
reduced aqueous tear secretion
bradycardia
hypotension
bronchospasm
sleep disturbance,decreased libido, fatigue, headache, dizziness
Which of the following is FALSE regarding laser trabeculoplasty versus surgical trabeculectomy
for treatment of glaucoma:
surgical trabeculectomy is more likely to cause cataract
surgical trabeculectomy is more likely to lower intra-ocular pressure significantly
the IOP-lowering effect of laser trabeculoplasty is more likely to be transient
surgical trabeculectomy is more likely to preserve visual acuity short-term
Your answer was CORRECT
Explanation
Surgical trabeculectomy is more likely to significantly lower intra-ocular pressure than laser
trabeculoplasty. However, visual acuity in the short-term is adversely affected with surgical
trabeculectomy compared to laser. This effect is due in large part to the development of cataract in
surgical trabeculectomy patients.
Dorzolamide lowers IOP by:
increasing uveoscleral outflow
increasing conventional (trabecular meshwork) outflow
decreasing aqueous production
decreasing episcleral venous pressure
Your answer was INCORRECT
Explanation
Dorzolamide is a topical carbonic anhydrase inhibitor that works in an analogous fashion to systemic
carbonic anhydrase inhibitors (acetazolamide, methazolamide) by inhibiting carbonic anhydrase on the
ciliary epithelium and thus decreasing aqueous production.
Dorzolamide's main side effects are:
burning or blurring on instillation
bitter or metallic taste in the mouth
It appears to produce far fewer systemic side effects than do the oral agents
All the following are predisposing factors for primary angle closure EXCEPT:
small lens size
anterior location of lens-iris diaphragm
small corneal diameters
short axial length
Your answer was CORRECT
Explanation
Primary angle closure is due to proximity of the peripheral iris to the cornea. This in turn is
facilitated by a large lens (occurs with age), a shallow AC, a small diameter cornea (which
narrows the angle), and a short axial length.
Which one of the following types of glaucoma is LEAST likely to respond to medical therapy
alone?
lens particle glaucoma
pigmentary glaucoma
pseudoexfoliation
phacolytic glaucoma
Your answer was INCORRECT
Explanation
Phacolytic glaucoma results when mature or hypermature cataracts leak soluble lens protein
through a visibly intact lens capsule. This heavy-molecular weight protein directly obstructs the
outflow of aqueous. Macrophages engorged with this material may also obstruct the outflow
channels. Although it is desirable to first bring the IOP under medical control by topical and
systemic therapy, definitive treatment requires removal of the lens.
Lens particle glaucoma may occur after cataract surgery with retained cortical material. The lens
material may resorb spontaneously. Medical management is used to control IOP; however,
surgical removal of the lens material may be necessary if medical management fails.
Lens-induced or phaco-anaphylactic glaucoma occurs when trauma and rupture of the lens
capsule causes an aggressive granulomatous inflammation with raised pressure
Which is TRUE regarding childhood glaucomas?
trabeculotomy is the preferred treatment of infantile glaucoma with cloudy corneas
spontaneous hyphaema in an infant is pathognomonic for juvenile xanthogranuloma
infantile glaucoma is usually unilateral
infantile glaucoma is often successfully controlled with medical therapy
Your answer was CORRECT
Explanation
Infantile glaucoma represents a developmental anomaly of the angle structures, with either an
intrinsic defect of the trabecular meshwork or mechanical obstruction of the TM by a membrane.
It is not amenable to long-term medical management, although aqueous suppressants are useful as
a temporizing measure before surgery.
Goniotomy requires a clear cornea for viewing of the needle knife. Trabeculotomy is the treatment
of choice with a cloudy cornea because an external incision is used and the trabeculotome is
rotated into the anterior chamber.
Juvenile xanthogranuloma (JXG) can cause spontaneous hyphemas. However, in any infant that
presents with spontaneous hyphema an ophthalmologist must consider child abuse. Other causes
of spontaneous hyphema include bleeding diatheses, leukemia, and retinoblastoma
All are TRUE of Possner-Schlossman syndrome EXCEPT:
long term follow-up is recommended
the irido-corneal angles are open
attacks are unilateral
up to 40% of patients are HLA-B5 positive
Your answer was INCORRECT
Explanation
Possner-Schlossman syndrome is characterized by recurrent attacks of unilateral acute elevation in
IOP associated with a mild anterior uveitis. The IOP is out of proportion to the degree of
inflammation. It is believed to be secondary to trabeculitis with Herpes virus believed to play a
role. Typical patients are young adults and males are more commonly affected than females. 40%
of patients are HLA-Bw54 positive. Patients should be followed up as a significant proportion will
develop chronic glaucomatous changes
A post-trabeculectomy sub-conjunctival injection of 5-FU is most safely administered:
temporally
180 degrees from the bleb
nasally
into the bleb
Your answer was INCORRECT
Explanation
There is wide variability in practice. However, conventionally, sub-conjunctival injections of 5-
FU in the post-operative period are administered 180 degrees from the bleb to minimise intra-
ocular diffusion and the risks of bleb leakage.
Which is FALSE regarding pigment dispersion syndrome:
peripheral iridotomy may reduce the process of pigment dispersion
exercise may cause the intra-ocular pressure to rise acutely
pupil block plays a role in the pathogenesis of pigment dispersion glaucoma
trabecular pigmentation is more uniform than in pseudo-exfoliation
Your answer was CORRECT
Explanation
Reverse pupil block - where the anterior chamber pressure is higher than the posterior chamber,
causing concavity of the iris - is believed to play an important role in the pathogenesis of PDS by
bringing the posterior iris surface into contact with the zonules. Conventional pupil block (iris
bombe) has no role in PDS. Peripheral iridotomy can flatten the iris contour and thereby reduce
the process of pigment chaffing; although the efficacy of laser iridotomy in the prevention of PDS
and subsequent pigmentary glaucoma is yet to be firmly established. Other options above are true
Prostaglandin analogues lower IOP predominantly by which one of the following mechanisms?
increased uveoscleral outflow
increased trabecular outflow
reduced vitreous volume
reduced aqueous production
Your answer was CORRECT
Explanation
The prostaglandin analogues lower IOP mainly by increasing uveoscleral outflow.
Selective laser trabeculoplasty is selective for which tissue?
pigmented trabecular meshwork
scleral spur
non-pigmented trabecular meshwork
ciliary body
Your answer was CORRECT
Explanation
Selective laser trabeculoplasty utilizes frequency-doubled Q-switched Nd:YAG laser, which
selectively targets melanin pigment in the trabecular meshwork cells. It is potentially safer than
ALT as there is no thermal or structural damage to non-targeted tissue.
All of the following are true of pigment dispersion syndrome EXCEPT:
iris transillumination and Krukenberg's spindle are consistent with the diagnosis
glaucoma where present tends to be more resistant to medical therapy than POAG
5% of patients develop pigmentary glaucoma
it is more common in myopes
Your answer was INCORRECT
Explanation
25-50% of patients with pigment dispersion develop pigmentary glaucoma. Other options are true
Brimonidine represents what pharmacological class of drug?
carbonic anhydrase inhibitor
alpha-2 agonist
parasympathomimetic
alpha-1 antagonist
alpha-1 agonist
Your answer was CORRECT
Explanation
Brimonidine is an alpha-2 agonist. Fluorophotometric studies suggest it has a dual mechanism of
action by reducing aqueous humour production and increasing uveoscleral outflow. It is contra-
indicated in patients taking monoamine oxidase inhibitors since it can potentiate a hypertensive
crisis. Its IOP lowering effects can be blunted by tricyclic antidepressants
Which index on the Humphrey visual field test provides an indication of the degree of generalised
loss in the visual field compared to age-adjusted normal population values?
mean deviation
corrected pattern standard deviation
pattern standard deviation
short-term fluctuation
Your answer was INCORRECT
Explanation
Mean deviation refers to the average deviation of sensitivity calculated from each test point from
age-adjusted normal population values, which provides an indication of the degree of generalised
field loss.
Pattern standard deviation represents a summary measurement of the average deviation of
individual test points from the normal hill of vision after correcting for any overall sensitivity
differences. If there is global reduction in sensitivity (from cataract for example) the pattern
standard deviation adjusts for this, to give a better indication of areas in the visual field that are
relatively less sensitive than other areas (useful for monitoring conditions such as glaucoma).
Corrected pattern standard deviation is the pattern standard deviation after taking into account the
short-term fluctuation (an indication of consistency of response assessed by testing ten pre-
selected points twice and analysing the difference).
Which topical IOP-lowering agent should be used with caution in corneal endothelial dysfunction?
apraclonidine
bimatoprost
timolol
dorzolamide
Your answer was INCORRECT
Explanation
Carbonic-anhydrase inhibitors can affect the corneal endothelial pump mechanism, and should
therefore be used in caution in patients with corneal endothelial dysfunction. There have been
several published reports
Which statement regarding red cell glaucoma is FALSE:
topical pilocarpine 2% is indicated in treatment
it is caused by mechanical obstruction of the trabecular meshwork
the size of hyphaema is an important prognostic indicator
topical dexamethasone is indicated in treatment
Your answer was CORRECT
Explanation
Miotics are contraindicated in the context of hyphaema as they can lead to pupil block - which is
more likely in hyphaema due to the possibility of clot formation around the pupillary margin.
Mydriatics are instead indicated. Other options above are true.
An 85-year old lady is referred to clinic via the opticians. She has a 1-week history of severe right-
sided headache with nausea and blurred vision. On examination, she has bilaterally shallow irido-
corneal angles with 180 degrees of irido-corneal touch on the right. Intra-ocular pressures are
16mmHg bilaterally. Her right cornea demonstrates Descemet's folds with stromal oedema. She
has right-sided keratitic precipitates, flare and glaukomflecken. The left eye is quiet. Optic discs
appear healthy in both eyes.
What is the diagnosis in the right eye?
primary angle closure
post-congestive angle closure
sub-acute angle closure
acute congestive angle closure
primary angle closure glaucoma
Your answer was INCORRECT
Explanation
This lady has post-congestive angle closure in the right eye. The headache, nausea, Descemet's
folds and glaukomflecken confirm a previous acute angle closure. The pressure is now normal
(hence post-congestive) due to either spontaneous re-opening of the angles (suggested by the fact
that there is irido-corneal touch in only 180 degrees) or due to ciliary body shut-down. She should
be treated with bilateral laser peripheral iridotomies and followed up for signs of glaucoma
progression. In the left eye, she has narrow angles which predispose her anatomically to angle
closure - that is, she has primary angle closure
Which condition is most likely to be associated with closed-angle glaucoma?
iridocorneal endothelial syndrome
Schwartz-Matsuo syndrome
topiramate use in a high hypermetrope
Fuch's heterochromic iridocyclitis
Your answer was CORRECT
Explanation
Figure: ICE syndrome
ICE syndrome is caused by proliferation and migration of abnormal corneal endothelial cells,
which grow over all structures in the anterior chamber. Growth of abnormal cells over the corneal
endothelium leads to corneal decompensation which frequently requires corneal grafting.
Meanwhile growth over the trabecular meshwork and iris leads to progressive closed-angle
glaucoma, which ensues in over 50% of ICE eyes.
Topiramate use can cause an idiosyncratic reaction whereby there is anterior rotation of the ciliary
body creating a secondary angle closure glaucoma. This is typically self-limiting once the
medication is stopped. This reaction is rare.
Schwartz-Matsuo syndrome is characterised by high IOP occurring in association with a
rhegmatogenous retinal detachment. Photoreceptor outer segments (+- RPE pigment) migrate
transvitreally into the aqueous, block the trabecular outflow pathways, and result in IOP elevation,
which is open-angle.
In Fuch's cyclitis, abnormal vessels can occur in the irido-corneal angle (leading to Amsler's sign,
where there is intra-operative AC haemorrhage during paracentesis) but these vessels do not lead
to synechialisation and chronic angle closure; rather the IOP elevation in Fuch's cyclitis is open-
angle.
Which is FALSE regarding betaxolol:
it may enhance optic disc micro-perfusion
it is a cardioselective beta blocker
it has a greater ocular hypotensive effect than timolol
it is conta-indicated in congestive cardiac failure
Your answer was CORRECT
Explanation
Betaxolol is a cardioselective beta blocker (that is, it has greater beta-1 effect on the myocardium
than beta-2 effect on the respiratory and ocular systems). It is the only cardioselective beta blocker
used in glaucoma. It has less ocular hypotensive effect than timolol but it has a potentially greater
effect on visual field preservation due to its effects on the microperfusion of the optic disc.
Abnormally heavy trabecular meshwork pigmentation is typically associated with all of the
following EXCEPT:
uveal melanoma
anterior segment dysgenesis
pigment dispersion
pseudoexfoliation
previous trauma
Your answer was CORRECT
Explanation
Heavy trabecular meshwork pigmentation is not a feature of anterior segment dysgenesis.
Heavy trabecular pigmentation suggests:
pseudoexfoliation
pigment dispersion
previous inflammation or surgery
uveal melanoma
Glaucomatous optic neuropathy is associated with damage to which types of retinal cells?
Amacrine cells
Bipolar cells
Photoreceptors
Ganglion cells
Your answer was CORRECT
Explanation
Glaucomatous optic neuropathy is caused by ganglion cell death.
Which class of IOP-lowering agents is chemically similar to sulphonamides?
alpha-2 agonists
prostaglandin analogues
beta-blockers
carbonic-anhydrase inhibitors
Your answer was CORRECT
Explanation
The carbonic anhydrase inhibitors are chemically related to the sulphonamides. CAI should
therefore be avoided in patients with a history of anaphylaxis or Stevens-Johnson syndrome after
sulphonamide administration.
A patient is commenced on a new glaucoma drop and at their 3-month review, they have an
intensely red eye and follicular conjunctivitis.
Which drop is most likely to blame?
bimatoprost
brimonidine
travaprost
timolol
Your answer was CORRECT
Explanation
Brimonidine (alphagan) is the most likely to cause a follicular conjunctivitis.
This question came in the FRCS (Glasgow) Part 2 in October 2014.
Which statement is FALSE regarding aqueous outflow:
the trabecular meshwork terminates anteriorly at Schwalbe's line
the cribriform trabecular meshwork occurs immediately beneath Schlemm's canal and has the
highest resistance of all meshwork layers
the trabecular meshwork cells maintain the state of hydration of the trabecular connective
tissue but have no phagocytic capacity
contraction of the anterior ciliary muscle increases aqueous outflow by its mechanical action on
the trabecular meshwork
Your answer was CORRECT
Explanation
The trabecular meshwork can be subdivided into three anatomical zones: the innermost uveal
meshwork, the corneoscleral meshwork and the outermost cribriform meshwork, which is adjacent
to Schlemm's canal and has the highest resistance to flow. The anterior ciliary muscle fibres are
attached to the trabecular meshwork and on contraction they open the meshwork, increasing
aqueous flow. Note that trabecular cells have phagocytic capacity in addition to maintaining the
health and hydration of the connective tissues.
Which of the following is the most characteristic gonioscopy finding in Possner-Schlossman
syndrome:
closed angle with iris bombe
open angle
Sampaolesi's line
closed angle with peripheral anterior synechae
Your answer was CORRECT
Explanation
Possner-Schlossman syndrome (PSS) is characterized by an open irido-corneal angle. This
differentiates the acute spike in intra-ocular pressure of PSS from angle-closure glaucoma and/or
inflammatory glaucoma.
All of the following statements are true regarding the initial presentation of primary congenital
glaucoma EXCEPT:
although surgical therapy is usually indicated, medical therapy is often used initially
gonioscopy usually has clearly identifiable landmarks facilitating goniotomy as a first line
therapy
horizontally-oriented breaks in Descemet's membrane may be found in the buphthalmic eye
photophobia and tearing may be the only presenting symptoms
Your answer was INCORRECT
Explanation
On initial visit, the cornea is often cloudy in congenital glaucoma, obscuring gonioscopy and
prohibiting a goniotomy. Carbonic anhydrase inhibitors and beta-blockers are used to lower IOP
initially, and after the cornea clears, a goniotomy or trabeculotomy may be performed. When
gonioscopy is possible, the normal landmarks are poorly recognizable, and some feel that a
Barkan's membrane overlies the angle. This membrane is incised at the time of goniotomy.
All statements are true of Schlemm's canal EXCEPT:
it is drained by collector channels and aqueous veins
it is lined by endothelium
it drains aqueous humour by the non-conventional pathway
it is a circumferential channel filled with aqueous humour
the aqueous from Schlemm's canal can join either the deep episcleral venous plexuses or drain
directly into superficial conjunctival veins
Your answer was CORRECT
Explanation
Drainage of aqueous humour via the Schlemm's canal is described as the conventional outflow
pathway and is responsible for 70-90% of aqueous outflow. The non-conventional pathway refers
to drainage of aqueous via the intercellular spaces between ciliary muscle fibres and the loose
connective tissue of the suprachoroidal space. Other options supplied above are true.
The termination of Descemet's membrane gives rise to:
the scleral spur
Schlemm's canal
Schwalbe's line
the pigmented trabecular meshwork
Your answer was CORRECT
Explanation
The termination of Descemet's membrane gives rise to Schwalbe's line, which is an important
landmark in gonioscopy.
In a patient with dense bilateral cataracts, which index on the Humphrey visual field is likely to be
most useful in monitoring for glaucoma progression?
short-term fluctuation
pattern standard deviation
mean deviation
false-negatives
Your answer was CORRECT
Explanation
Mean deviation refers to the average deviation of sensitivity calculated from each test point from
age-adjusted normal population values, which provides an indication of the degree of generalised
field loss.
Pattern standard deviation represents a summary measurement of the average deviation of
individual test points from the normal hill of vision after correcting for any overall sensitivity
differences. If there is global reduction in sensitivity (from cataract for example) the pattern
standard deviation adjusts for this, to give a better indication of areas in the visual field that are
relatively less sensitive than other areas (useful for monitoring conditions such as glaucoma).
Corrected pattern standard deviation is the pattern standard deviation after taking into account the
short-term fluctuation (an indication of consistency of response assessed by testing ten pre-
selected points twice and analysing the difference).
Medical management of bleb leaks involves all of the following EXCEPT:
aqueous suppressants
autologous blood injection
bandage soft contact lens
5-fluorouracil
Your answer was CORRECT
Explanation
5-FU is used to prevent (and/or treat) post-operative episcleral fibrosis, and has no role in the
treatment of bleb leaks. Indeed anti-metabolite use is a risk factor for bleb leaks.
All of the following clinical features may lead to a suspicion of plateau iris in a patient with angle
closure EXCEPT?
a convex iris plane
deep anterior chamber centrally
a young patient with myopia
an anteriorly located ciliary body on OCT imaging
Your answer was CORRECT
Explanation
The iris plane in plateau iris syndrome is flat rather than convex. A convex plane is observed in
conventional pupil-block angle-closure. Other options supplied above are true.
Which of the following is NOT found in pseudoexfoliation:
peripheral iris transillumination
Krukenberg spindle
hyperpigmentation of trabecular meshwork
Sampaolesi line
Your answer was INCORRECT
Explanation
Iris transillumination defects are found in pseudo-exfoliation but they typically occur around the
pupillary sphincter, with a classic moth-eaten appearance. Other options are true.
Note: While Krukenberg spindle and Sampaolesi line are more commonly associated with
pigment dispersion syndrome, they are both recognised features of pseudo-exfoliation syndrome.
There are many sources to confirm this, e.g. Ariga M et al. Pseudoexfoliation Syndrome. J Curr
Glaucoma Pract. 2013 Sep-Dec; 7(3): 118–120.
All of the following are viable treatment options for angle-recession glaucoma EXCEPT:
topical beta blockade
an artificial filtering shunt
topical prostaglandin analogues
laser trabeculoplasty
Your answer was CORRECT
Explanation
Angle recession is treated initially as for any chronic open-angle glaucoma with topical IOP-
lowering agents. If these fail, surgical trabeculectomy with adjunctive anti-metabolites or an
artificial filtering shunt can be considered. Note: laser trabeculoplasty is ineffective in angle
recession.
All are true of pigment dispersion syndrome EXCEPT:
it is usually bilateral
it is more common in whites than blacks
myopia increases the risk
convexity of the mid-peripheral iris is believed to increase the risk
Your answer was CORRECT
Explanation
Concavity of the mid-peripheral iris, as occurs in myopes and long axial length eyes, increases the
risk of pigment dispersion because the posterior iris surface comes into contact with the zonules.
Mechanical friction then causes the release of pigment from the iris into the anterior and posterior
chambers. Other options supplied above are true.
Which is FALSE regarding intra-ocular pressure?
it is not affected by posture in healthy eyes
it always displays a diurnal rhythm
it increases with age
it is increased in extreme positions of gaze
Your answer was INCORRECT
Explanation
IOP is dependent on posture in both healthy and glaucomatous eyes. Several studies have shown,
for example, that IOP is lower when a person sleeps inclined head up, compared to lying flat
(reference). Other options supplied above are true.
Secondary angle-closure glaucoma may be associated with each of the following conditions
EXCEPT:
scleral buckle for retinal detachment
Schwartz-Matsuo syndrome
intraocular tumor
nanophthalmos
Your answer was CORRECT
Explanation
Schwartz-Matsuo syndrome is the name given to open-angle glaucoma after rhegmatogenous
retinal detachment. The elevated IOP results from obstruction of outflow by inflammation,
pigment released from the retinal pigment epithelium (RPE), glycosaminoglycans released by the
photoreceptors, or photoreceptor outer segments. It usually resolves after repair of the retinal
detachment (RD).
Scleral buckles that are too tight can cause ciliary body effusions, forward rotation of the ciliary
body and secondary angle-closure glaucoma.
Nanophthalmic eyes are at risk of uveal-effusion syndrome due to thick sclera impeding vortex
vein drainage. This can cause ciliary body effusions, which can lead to a secondary angle-closure
glaucoma.
All of the following conditions are associated with elevated IOP except:
idiopathic intracranial hypertension
carotid-cavernous fistula
juvenile idiopathic arthritis
thyroid eye disease
Your answer was CORRECT
Explanation
Idiopathic intracranial hypertension is caused by raised intracranial pressure which can lead to
papilloedema, with subsequent optic neuropathy but it is not associated with elevated IOP per se.
Which of the following glaucoma drops decreases aqueous production AND increases aqueous
outflow?
bimatoprost
betoxolol
dorzolamide
brimonidine
Your answer was CORRECT
Explanation
Alpha-2 adrenergic agonists such as brimonidine work by a dual mechanism, decreasing aqueous
production and increasing outflow. Dorzolamide is a carbonic anhydrase inhibitor, which
decreases aqueous production. Prostaglandin analogues such as bimatoprost increase aqueous
outflow by the uveo-scleral route. Beta-blockers such as betoxolol reduce aqueous production.
This question appeared in the 2014 FRCOphth Part 2.
Which topical beta-blocker is LEAST likely to cause bradycardia:
carteolol
betaxolol
metipranolol
timolol
Your answer was CORRECT
Explanation
Carteolol has intrinsic sympathomimetic activity, thus inducing less bradychardia and
bronchospasm than timolol.
Which portion of the iridocorneal angle is usually the easiest for distinguishing landmarks on
gonioscopy?
nasal
inferior
superior
temporal
Your answer was CORRECT
Explanation
The inferior angle (viewed through the superior mirror on a Zeiss gonioprism lens) is usually the
widest and is generally considered the easiest portion for distinguishing landmarks.
Which treatment is most likely to be effective in controlling ICE syndrome glaucoma:
surgical trabeculectomy with adjuvant antimetabolites
glaucoma drainage device
laser peripheral iridotomy
laser trabeculoplasty
Your answer was INCORRECT
Explanation
Glaucoma is difficult to control in ICE syndrome, even with surgical trabeculectomy which is
prone to bleb failure. Artificial filtering shunts are the definitive treatment and are required for
effective pressure control in many cases.
A 38-year-old woman presents with unilateral glaucoma. Examination reveals pseudopolycoria
and peripheral anterior synechae. The inner cornea has an odd appearance.
What is the most likely diagnosis?
Epithelial downgrowth
Post-trauma with angle-recession glaucoma
Axenfeld-Rieger syndrome
Chandler syndrome
Cogan-Reese syndrome
Your answer was INCORRECT
Explanation
This patient has irido-corneal endothelial syndrome, which may present as 3 variants: Cogan-
Reese syndrome (iris naevi), essential iris atrophy (mainly iris abnormalities such as
pseudopolycoria) and Chandler’s syndrome, in which there are corneal endothelial abnormalities
in addition to iris abnormalities. ICE is usually unilateral.
The following are true about the drugs used to modulate wound healing post glaucoma surgery,
EXCEPT:
MMC is an alkylating agent that decreases DNA synthesis by causing DNA cross-linking
BAPN is an inhibitor of lysyl oxidase and blocks collagen cross-linking
5-FU inhibits fibroblast proliferation by acting selectively on the S phase of the cell cycle
colchicine acts by inhibiting fibroblast migration and proliferation
Your answer was INCORRECT
Explanation
Colchicine affects collagen cross-linking and thereby decreases scar formation. All other options
supplied are true.
Initial treatment options for primary congenital glaucoma include all of the following EXCEPT:
goniotomy
trabeculotomy
medical therapy
trabeculectomy
Your answer was INCORRECT
Explanation
Congenital glaucoma is a surgical disease. The basic abnormality is a localized dysgenesis of the
superficial angle structures, and both trabeculotomy and goniotomy may alleviate this problem.
IOP-lowering medications are, however, useful in the management of pediatric glaucoma: they
may clear corneal oedema, which in turn facilitates surgery; the lower IOP may lessen optic nerve
damage until surgery can be performed; and the IOP-lowering medications may be used in the
postoperative course if additional lowering of IOP is required. Trabeculectomy may be an option,
but given that these young patients tend to heal exuberantly, its success may be limited and it (plus
or minus tubes) is usually only considered after attempts at goniotomy and trabeculotomy have
been attempted.
An episode of intermittent sub-acute angle closure is most likely to be terminated by:
coughing
sleep
semi-prone position
emotional stress
Your answer was INCORRECT
Explanation
An episode of sub-acute angle closure is most likely to be broken by physiological miosis - for
example due to exposure to bright light or sleep. Semi-prone position and emotional stress are
likely to precipitate angle closure in the first place. Coughing and the Valsalva maneuver in
general have no relation to acute angle-closure glaucoma.
Which of the following drugs is a direct agonist that increases aqueous flow by the conventional
route?
pilocarpine
latanoprost
apraclonidine
echothiophate
Your answer was CORRECT
Explanation
Pilocarpine is a direct-acting cholinergic agonist, which reduces IOP by increasing aqueous
outflow through the trabecular meshwork (i.e. the conventional route). Echothiphate is an indirect-
acting cholinergic agonist which irreversibly inhibits acetylcholinesterase. Latanoprost reduces
IOP by increasing outflow via the uveo-scleral route (alternative pathway). Apraclonidine is
primarily an alpha-2 agonist (with some alpha-1 activity), which lowers IOP by reducing aqueous
secretion.
This question appeared in the 2014 FRCOphth Part 2.
Which topical IOP-lowering agent has limited use due to tachyphylaxis:
apraclonidine
unoprostone isopropyl
brimonidine
metipranolol
Your answer was CORRECT
Explanation
Apraclonidine (iopidine) is mainly used after surgery on the anterior segment to prevent a sudden
rise in intra-ocular pressure. Its routine longterm use for IOP control is limited by tachyphylaxis -
that is, a lowering of its effect after ongoing use. About 10% of patients using beta-blockers also
get tachyphylaxis (either within a few days or a few months), but it is not as prominent as
apraclonidine.
A 78-year-old African American man had a cataract extraction with posterior chamber IOL in the
right eye 7 years ago, and 5 years ago he developed a retinal detachment in that eye, which was
repaired with a scleral buckle. He developed intractable glaucoma in that eye with an open angle
on gonioscopy. Despite having undergone two sessions of laser trabeculoplasty and now being on
maximum-tolerated medical therapy with a visual acuity of 20/200, the IOP remains at 28 mmHg.
Slit-lamp examination reveals 360 degree of scarred and non-mobile conjunctiva, and there is
almost total cupping of the nerve.
Which one of the following procedures would be MOST appropriate at this point?
transscleral cyclophotocoagulation
trabeculectomy with antimetabolite
laser trabeculoplasty
laser iridotomy
Your answer was CORRECT
Explanation
This gentleman is at high risk of bleb failure with trabeculectomy due to Afro-Caribbean race,
multiple previous surgeries and scarred conjunctiva. If a drainage procedure is being considered
then it would have to be a glaucoma tube (not provided in the options). Given that the vision in
this eye is so poor from previous retinal detachment, drainage surgery is probably too involved in
any case, and a more suitable option is transcleral cyclodiode laser.
The adjunctive use of anti-fibrotic agents in trabeculectomy is indicated in all of the following
situations EXCEPT:
aphakic patients
previously failed filtering surgery
young myopic patients
neovascular glaucoma
Your answer was INCORRECT
Explanation
Although originally advocated for use in high risk eyes such as those with aphakia, pseudophakia,
neovascular glaucoma, or a history of previously failed surgeries, anti-fibrotic agents are now
routinely used by many surgeons. Anti-fibrotic agents should be used, however, with caution in
young myopic patients due to the risk of hypotony.
Which statement regarding intraocular pressure is FALSE?
the peak IOP in most individuals occurs during the day
glaucomatous eyes have greater diurnal variation
normal IOP diurnal fluctuation ranges from 2 to 6 mmHg
intraocular pressure is distributed along a normal Gaussian curve
Your answer was CORRECT
Explanation
The distribution of intraocular pressure (IOP) cannot be predicted by the normal (gaussian)
distribution. The use of the mean +/-2 standard deviations as the range of normal IOP in the
general population is not valid because the actual distribution is skewed toward higher IOP.
A 58-year-old patient with chronic obstructive airways disease attends his optician routinely. He
has no visual complaints but IOP is 32mmHg bilaterally on pneumatic tonometry. Which inhaled
medication is most likely to be responsible?
salbutamol
saline nebuliser
ipratropium
fluticasone
Your answer was CORRECT
Explanation
Steroid inhalers can increase IOP in steroid responders. Up to 5% of the general population are
steroid responders, with an elevation over 15mmHg from baseline. While beta-agonists such as
salbutamol and anti-cholinergics such as ipratropium may induce angle-closure by causing
mydriasis, this is a rare event, and the patient is likely to be symptomatic.
This question appeared in the 2014 FRCOphth Part 2
Which is TRUE of pseudo-exfoliation?
it may be associated with increased risk of cardiovascular disease
it may be associated with pressure spikes following exercise
it is most common in the fifth decades
it is more common in Japanese
Your answer was CORRECT
Explanation
Pseudo-exfoliation is a disease of the elderly and incidence increases with each decade. It is most
prevalent in Finland and Norway, while incidence in Japan is much lower. It is a systemic disease
and some studies have suggested a risk of cardiovascular diseases such as myocardial infarction
and stroke.
Note: a pressure spike following exercise is a feature of pigment dispersion syndrome, not pseudo-
exfoliation.
This question came in the FRCS (Glasgow) Part 2 in October 2014.
Which sub-type of glaucomatous nerve damage is characterized by a shallow saucerised cup and a
gently sloping rim?
senile sclerotic
concentrically enlarging
focal ischaemic
myopic
Your answer was CORRECT
Explanation
The sub-types of glaucomatous optic nerve damage are:
Type 1 (focal ischaemic): characterized by a focal notch in the neuroretinal rim. It usually occurs in
females with a history of migraine or vasospasm. It is typically associated with a localised field
defect near to fixation.
Type 2 (myopic glaucomatous): associated with a temporal crescent in the absence of degenerative
myopia. It is associated with dense superior or inferior field loss and tends to occur in younger male
patients.
Type 3 (senile sclerotic): characterised by a shallow saucerised cup and a gently sloping rim. As its
name suggests, it typically occurs in elderly patients and is associated with hypertension and
cardiovascular disease.
Type 4 (concentrically enlarging): characterised by thinning of the entire rim in the absence of
notching. It is associated with a high IOP at presentation and tends to occur in younger paints. Field
loss is frequently diffuse.
Type 5: mixed
Use of 5-FU during glaucoma filtration surgery has been associated with all of the following
complications EXCEPT:
conjunctival wound leaks
retinal detachment
hypotonous maculopathy
corneal epithelial toxicity
suprachoroidal hemorrhage
Your answer was CORRECT
Explanation
Side-effects of 5-FU include:
conjunctival wound leaks
corneal epithelial defects
thin-walled ischaemic blebs
hypotony
suprachoroidal hemorrhage
It has not been associated with increased risk of retinal detachment.
The eye condition LEAST likely to be associated with aqueous misdirection syndrome is:
uveitis
angle-closure glaucoma
nanophthalmos
pigment dispersion syndrome
Your answer was INCORRECT
Explanation
Pigment dispersion is not a risk factor for aqueous misdirection.
Risk factors for aqueous misdirection include:
angle closure eyes
nanophthalmic eyes
ciliary body inflammation e.g. trauma, surgery
A 31-year-old myope presents to ophthalmology casualty with blurred vision and moderate
photophobia. On examination, his IOP is 50mmHg. What is the most likely diagnosis?
Posner-Schlossman syndrome
acute closed angle glaucoma
herpetic keratitis
toxoplasma uveitis
Your answer was CORRECT
Explanation
All the options above can cause pain and raised IOP. Acute angle closure glaucoma is, however,
unlikely in a myope and would be associated usually with greater pain. Herpetic keratitis and
toxoplasma uveitis would be unlikely to cause such an elevated IOP on first presentation in the
absence of greater inflammation. Posner-Schlossman syndrome is the most likely. It causes raised
IOP, often accompanied by blurred vision secondary to mild corneal oedema. There is typically a
mild iritis.
This question came in the 2014 FRCOphth.
The MOST important treatment for a patient with diabetic neovascular glaucoma and good vision
is:
panretinal photocoagulation
adequate blood sugar and blood pressure control
cyclophotocoagulation
aqueous suppressants
atropine and topical steroids
Your answer was CORRECT
Explanation
PRP laser is the most important treatment, as this targets the source of the problem by reducing
VEGF production from ischaemic retina. New vessels may regress within a few days to a few
weeks.
Which vessel supplies the majority of blood to the surface nerve fiber layer of the optic nerve
head?
Peripapillary choroidal vessels
Pial vessels
Short posterior ciliary artery
Central retinal artery
Your answer was CORRECT
Explanation
The four divisions of the optic nerve head correlate roughly with a four-part blood supply:
1. The surface nerve fiber layer is supplied mainly by branches of the central retinal artery.
2. The prelaminar region is supplied by capillaries of the short posterior ciliary arteries.
3. The lamina cribrosa region is also supplied by vessels that come directly from the short
posterior ciliary arteries to form a dense plexus in the lamina.
4. The retrolaminar region is supplied by both the ciliary and retinal circulations, with the former
coming from recurrent pial vessels, while the central retinal artery provides centripetal branches
from the pial region
All are true regarding argon laser trabeculoplasty (ALT) EXCEPT:
a heavily pigmented trabecular meshwork requires greater laser energy
it enhances trabecular outflow
approximately 50 burns are created over 180 degrees of the meshwork
6 weeks should be allowed for IOP-lowering to take effect
Your answer was CORRECT
Explanation
A heavily pigmented trabecular meshwork requires less laser energy than a lightly pigmented one.
Other options above are true.
The average initial reduction in IOP from argon laser trabeculoplasty is:
10%
30%
5%
20%
Your answer was CORRECT
Explanation
The average initial reduction in intra-ocular pressure from ALT is 30%. ALT is successful at
reducing IOP in 75-85% of patients. However the effect is often short-term. At 5 years, the
success rate is 50% and at 10 years the success is 33%. Early failure is more common in black
patients, young patients, and patients with pseudoexfoliation compared to primary open-angle
glaucoma. Paediatric glaucomas and most secondary glaucomas do not respond well to ALT. Re-
treatment following initial success is less likely to succeed with a success rate of only 30%.
Renal stone formation is a recognized side-effect of which class of IOP-lowering agent:
beta-blockers
carbonic-anhydrase inhibitors
alpha-2 agonists
prostaglandin analogues
Your answer was CORRECT
Explanation
Carbonic anhydrase inhibitor (CAI) side-effects include:
Paraesthesia of fingers, toes, hands, feet, mucocutaneous junctions
Malaise complex: fatigue, depression, weight loss, reduced libido
GI complex: diarrhea, nausea, cramps
Renal stone formation
Steven-Johnson syndrome
Blood dyscrasias: bone marrow suppression, aplastic anaemia.
2 days post-trabeculectomy a patient has a shallow anterior chamber with an intra-ocular pressure
of 18mmHg. What is the most likely cause?
bleb leak
aqueous misdirection
ciliary body shutdown
uveitis
Your answer was INCORRECT
Explanation
The most likely scenario is aqueous misdirection. Aqueous is directed posteriorly, causing
shallowing of the anterior chamber, flat bleb, and a relatively elevated IOP. Ciliary body
shutdown and bleb leak would cause a shallow anterior chamber with hypotony. Uveitis on its
own cannot explain anterior chamber shallowing.
This question appeared in the 2014 FRCOphth Part 2.
Acetazolamide is contra-indicated in which scenario:
allergy to gentamicin
allergy to benzylpenicillin
allergy to sulfamethoxazole
allergy to ciprofloxacin
Your answer was CORRECT
Explanation
Acetazolamide is a carbonic anhydrase inhibitor and part of the sulfonamide class of diuretics,
which include thiazide diuretics, thiazide-like diuretics and loop diuretics. As a sulfonamide drug,
acetazolamide is structurally related to the sulfonamide group of antibiotics, and to the
sulfonylurea diabetic agents such as glipizide. A severe allergy to a sulfa- antibiotic is therefore a
contraindication to the use of acetazolamide.
This question came in the 2014 FRCOphth.
All of the following are risk factors for blebitis EXCEPT:
supero-temporal bleb
blepharitis
bleb leak
use of anti-metabolites
Your answer was CORRECT
Explanation
Risk factors for blebitis include:
thin-walled bleb
bleb leak
long-term topical antibiotics
inferior or nasal bleb
blepharitis
Apraclonidine drops for glaucoma are LEAST likely to cause which side-effect:
blanching of conjunctival vessels
dry mouth
superior lid retraction
systemic hypotension
Your answer was INCORRECT
Explanation
Apraclonidine hydrochloride is an alpha-2-adrenergic agonist that is used clinically as a potent systemic anti-
hypertensive agent. Several studies, however, have shown the lack of effect of apraclonidine in drop form on
blood pressure and pulse. It does, however, cause a transient dry mouth or dry nose.
Ocular side effects of apraclonidine include:
eyelid retraction
mydriasis
conjunctival blanching
All of the following are recognised methods for long-term management of infantile glaucoma
EXCEPT:
goniotomy
trabeculotomy
trabeculectomy
oral carbonic anhydrase inhibitors
Your answer was CORRECT
Explanation
Medication is only a temporising measure in the management of infantile glaucoma, used for
preoperative control of IOP. Definitive management consists of surgery.
A 75-year-old lady recently changed her topical glaucoma medication and reports that since the
change her previously droopy eyelids have improved.
What is the most likely treatment she is currently on?
brinzolamide
bimatoprost
propine
brimonidine
timolol
Your answer was CORRECT
Explanation
Brimonidine is an alpha-2 agonist. It stimulates the Muller muscle, thus improving ptosis.
Which drug used during general anaesthesia is associated with an increase in intraocular pressure?
Phenobarbital
Valium
Ketamine
Halothane
Your answer was CORRECT
Explanation
In most cases, patients have a decrease in IOP with general anaesthesia. In particular, halothane
and the inhalational anesthetics can decrease IOP. Ketamine, trichloroethylene and
succinylcholine, however, can cause IOP to increase. Tranquilizers and barbiturates may cause a
slight decrease in IOP.
In the context of paediatric glaucoma, ketamine has traditionally been the anaesthetic of choice
because it does not lead to a falsely low IOP reading.
Gonioscopy in primary congenital glaucoma is likely to reveal:
absence of the ciliary body band
absence of the trabecular meshwork
absence of the scleral spur band
wide angles with peripheral anterior synechae beyond Schwalbe's line
Your answer was INCORRECT
Explanation
Primary congenital glaucoma is caused by mal-development of the angle of the anterior chamber,
unassociated with any other major ocular anomalies (isolated trabeculodysgenesis). This is
charactereised clinically by the absence of the ciliary body band on gonioscopy due to translucent
amorphous material that obscures the trabeculum.
Which of the following statements about pigmentary glaucoma is FALSE?
there is an association with myopia
women affected by the disease are typically younger than affected men
radial iris defects are more specific for the condition than Krukenberg spindles
intraocular pressure may fluctuate widely in this disorder
Your answer was CORRECT
Explanation
The sine qua non of pigment dispersion syndrome is radial defects in the iris pigment epithelium.
Krukenberg spindles are less specific. Pigmentary glaucoma usually occurs in young myopic men,
typically in their third or fourth decade of life. For obscure reasons, women with the disease tend
to be older then men. Exercise or pupillary movements may induce a shower of iris pigment
release, with resultant increased intraocular pressure
Iridocorneal endothelial (ICE) syndrome glaucoma is caused by:
reverse pupil block angle closure
synechial angle closure
open angle trabecular obstruction
pupil block angle closure
Your answer was INCORRECT
Explanation
ICE syndrome is caused by an abnormal corneal endothelial cell layer which proliferates and
migrates across the angle and onto the iris. Glaucoma occurs in about 50% of cases, due to
synechial angle closure secondary to contraction of this abnormal tissue. PCR suggests a possible
herpes simplex viral origin to the disease process.
A 53-year-old white woman has glaucomatous optic nerve head changes in both eyes and split
fixation in her right eye, consistent with her disc findings. She has progressively lost visual field
and neural rim tissue while running intraocular pressures in her early teens. Gonioscopy is normal
with open angles. She is currently on maximal tolerated medical therapy and reports subjective
decrease in vision in her right eye.
What is the most appropriate next step?
laser iridotomy
trabeculectomy
glaucoma tube drainage device
cyclophotocoagulation
iridoplasty
Your answer was CORRECT
Explanation
Given the extent of the glaucomatous damage as indicated by split fixation and progression of
both fields and disc at low-normal pressures, the maximal decrease in IOP possible is necessary.
Primary filtration presents the best choice. Iridoplasty is usually reserved for eyes with plateau iris
syndrome. Peripheral iridectomy would not be appropriate because the angles appear normal.
Cyclophotocoagulation is reserved for end-stage disease. Drainage devices such as the Molteno,
Baerveldt, and Ahmed are reserved for special cases where a high rate of bleb failure with
standard trabeculectomy is anticipated.
All are true of glaucoma secondary to Sturge-Weber syndrome EXCEPT:
it is usually unilateral
most patients who develop glaucoma do so after the age of 2 years
trabecular dysgenesis is important in the pathogenesis of early-onset glaucoma
raised episcleral venous pressure is important in late-onset glaucoma
Your answer was CORRECT
Explanation
Most patients with Sturge-Weber syndrome who develop glaucoma do so before the age of 2
years.
In Sturge-Weber syndrome, isolated trabeculodysgenesis is important in the pathophysiology of
early-onset glaucoma. Meanwhile, raised episcleral venous pressure is important in late-onset
glaucoma.
All of the following anaesthetic agents diminish intraocular pressure EXCEPT?
isoflurane
enflurane
halothane
ketamine
Your answer was CORRECT
Explanation
Ketamine is the preferred anaesthetic for measuring intraocular pressure because it does not lower
IOP like the halothane agents. In fact, ketamine can if anything cause an increase in IOP,
primarily at high doses. Succinylcholine also can elevate IOP.
All are true regarding argon laser trabeculoplasty (ALT) EXCEPT:
the laser should be directed at the junction of the pigmented and non-pigmented meshwork
the success rate of subsequent filtration surgery is compromised by ALT
the initial success rate of ALT is about 75% in POAG
prophylactic apraclonidine or brimodine should be used at treatment
Your answer was INCORRECT
Explanation
The AGIS study (AJO, 2002, 134:481-98) found no evidence to support the assertion that
trabeclectomy success is compromised by prior ALT. Factors associated with trabeclectomy
failure in AGIS were: younger age, higher pre-intervention IOP, diabetes, and one or more
postoperative complications, particularly elevated IOP and marked inflammation.
All of the following slit-lamp findings are consistent with a diagnosis of primary angle closure
EXCEPT:
close proximity of iris to cornea peripherally
optic disc pallor and cupping
convex iris-lens diaphragm
shallow anterior chamber
Your answer was CORRECT
Explanation
Primary angle closure (PAC) refers to an anatomically predisposed eye. One might expect to find
a shallow chamber, convex iris-lens diaphragm and occludable angles on gonioscopy. However,
there is no optic disc cupping. Disc cupping is a sign of primary angle closure glaucoma (PACG).
Glaukomfleken is best described as:
polychromatic opacities within the lens nucleus
small, white opacities beneath the anterior capsule of the lens
spoke-like radial opacities beneath the anterior capsule
small, fleckled, posterior sub-capsular opacities
Your answer was CORRECT
Explanation
Glaukomfleken are small, white opacities beneath the anterior capsule of the lens that are
pathognomonic for previous acute angle-closure.
Which is TRUE regarding intra-ocular pressure and glaucoma?
angle-closure following scleral buckling is best relieved by peripheral iridotomy
angle-closure is more common in males
digital pressure on a Goldmann gonioscopy lens tends to open the angle
the Perkins tonometer is accurate on both erect and supine patients
Your answer was CORRECT
Explanation
The Perkins tonometer is counterbalanced so that it can be used in the erect or supine position.
The Tonopen, a portable electronic applanation device, can also be used in the erect or supine
position.
Digital pressure on a Goldmann lens (which is large) tends to shallow the angle, while pressure on
a smaller Zeiss lens tends to open the angle.
Peripheral iridotomy does not help relieve the glaucoma associated with scleral buckling because
the mechanism of angle closure is not pupillary block. Obstruction of venous outflow produces
choroidal effusions that cause anterior rotation of the ciliary body and secondary angle closure.
Another mechanism of angle closure following buckling surgery is neovascular glaucoma caused
by buckle-induced ocular ischemia. If medical management is not effective during the acute
period, the buckle may need to be repositioned or removed.
A mother brings in her 6-month-old son for evaluation of excessive tearing from both of his eyes.
On examination, the child is noted to be photophobic and have corneal clouding.
What is the best course of action to take?
perform B-scan ultrasonography
examine the patient under general anaesthesia
refer to a paediatric neurologist for consideration of brain MRI
discharge with advice on nasolacrimal sac massage
Your answer was CORRECT
Explanation
The presentation is suspicious for bilateral infantile glaucoma. If possible, the diagnosis may be
made in the office through IOP measurement, cycloplegic refraction (induced myopia), and optic
nerve examination. In most cases, however, the patient needs to be taken to the operating room to
confirm the diagnosis and to initiate surgical correction while under the same anaesthesia.
Initial treatment of iridoschisis involves:
laser trabeculoplasty
surgical trabeculectomy with adjuvant antimetabolites
peripheral iridotomy
topical IOP-lowering medication
Your answer was INCORRECT
Explanation
Iridoschisis is thought to result from intermittent angle-closure, which causes iris atrophy and
schisis due to the high pressure. Findings include shallow anterior chamber with occludable angles
on gonioscopy. Iridoschisis usually involves the inferior iris. Initial treatment is with peripheral
iridotomy.
Which statement is TRUE regarding the treatment of pigment dispersion glaucoma:
laser trabeculoplasty will require a higher initial laser setting
men tend to require trabeculectomy earlier than women
a lower percentage of patients require trabeculectomy than POAG
treatment with miotics is contraindicated
Your answer was CORRECT
Explanation
Pigmentary glaucoma can often be treated effectively with miotics, as they flatten the iris and
prevent iris chaffing. However, miotics are often poorly tolerated by patients, especially myopes
who make up the majority of PDS patients. Due to the heavily pigmented trabecular meshwork,
laser trabeculoplasty should be performed with a low initial laser setting. Men with PDS are more
likely to develop glaucoma and are more likely to require surgical trabeculectomy. A higher
percentage of pigmentary glaucoma patients require trabeculectomy than POAG.
Which of the following is NOT a common side-effect of topical prostaglandin analogues?
eyelash hypertrichosis
cystoid macular oedema
iris atrophy with iris transillumination defects
conjunctival hyperaemia
darkening of periocular skin
Your answer was CORRECT
Explanation
Iris atrophy is not a recognised side-effect of the prostaglandin analogues. They can, however,
cause iris hyperpigmentation and iris pigment epithelial cysts.
All are true of steroid-induced glaucoma EXCEPT:
the rise in IOP may be delayed for years after starting the steroid
a family history of glaucoma is a risk for steroid-induced IOP elevations
it is more likely with oral steroids than topical steroids
after discontinuing steroids IOP typically returns to normal in a few days to several weeks
Your answer was CORRECT
Explanation
Oral steroids are more likely to cause cataract than topical steroids, but they are less likely to
cause steroid-induced raised intraocular pressure.
Which is FALSE regarding prostaglandin analogues used in glaucoma:
uveitis and CMO are rare complications
latanoprost is a pro-drug that becomes active after being hydrolysed by corneal esterase
bimatoprost is a prostamide analogue
they can cause lash hypertrichosis and peri-orbital fat atrophy
travaprost increases both uveoscleral and conventional outflow
Your answer was INCORRECT
Explanation
Latanoprost and travaprost are both pro-drugs that become activated when they are hydrolysed by
corneal esterases. The are both prostaglandin F agonists which lower IOP by increasing
2α
uveoscleral outflow. Bimatoprost is structurally similar but has a slightly different mode of action.
Bimatoprost is a prostamide analogue which lowers IOP by increasing both uveoscleral and
trabecular outflow. Other statements above are true.
Which beta-blocker has intrinsic sympathomimetic activity?
betoxolol
levobunolol
timolol
carteolol
Your answer was CORRECT
Explanation
Of the glaucoma options, carteolol is the only beta-blocker that has intrinsic sympathomimetic
activity, so will induce less bradycardia and postural hypotension than other options.
Which statement regarding aqueous humour is FALSE?
the anterior chamber volume is approximately 200 to 250 microlitres
the rate of aqueous formation is approximately 2 to 3 microlitres per minute
uveoscleral outflow is pressure-dependent
aqueous production decreases with age, trauma and inflammation
miotics decrease uveoscleral outflow facility
Your answer was INCORRECT
Explanation
Uveoscleral outflow is pressure-independent. Note that miotics lower intraocular pressure by
increasing trabecular outflow facility (conventional outflow pathway); by contradiction, miotics
actually decrease uveoscleral outflow (unconventional outflow pathway). Cycloplegic agents,
epinephrine, and prostaglandin analogues have been shown to increase uveoscleral flow.
Which of the following topical IOP-lowering agents is LEAST likely to be indicated in the context
of uveitis-related raised IOP?
carbonic anhydrase inhibitors
beta-blockers
prostaglandin analogues
alpha-adrenergic agonists
Your answer was CORRECT
Explanation
Prostaglandin analogues are generally avoided in uveitis-related ocular hypertension/glaucoma
due to the small risk of precipitating a uveitic episode or cystoid macular oedema (CMO).
Miotics are also avoided due to the fact that they increase vascular permeability and may therefore
induce inflammation, as well as worsen posterior synechae.
Beta-blockers are generally the first line, with consideration given to adding an alpha-adrenergic
agonist or carbonic anhydrase inhibitor if the pressure is not adequately controlled.
Primary congenital glaucoma (PCG) affects:
1:1,000,000 live births
1:10,000 live births
1:100,000 live births
1:1000 live births
Your answer was CORRECT
Explanation
PCG affects 1:10,000 live births. 65% of cases are male
All of the following are true of visual field changes in glaucoma EXCEPT:
the most common early field defect is a paracentral defect respecting the horizontal midline
paracentral scotomas occur most commonly infero-nasally
end-stage changes are usually characterised by a central island and a temporal island of vision
arcuate scotomas typically develop from coalescence of paracentral scotomas
Your answer was INCORRECT
Explanation
Paracentral scotomas are the most common early glaucomatous field defect, constituting 70% of
all early defects. They are usually supero-nasal and respect the horizontal midline. Other options
above are true.
A 6-month-old child is diagnosed with bilateral primary congenital glaucoma. Which of the
following is the LEAST likely cause of vision loss:
optic nerve damage
corneal scarring
myopic astigmatism
anisometropic amblyopia
Your answer was CORRECT
Explanation
Most cases of primary congenital glaucoma are bilateral (over two-thirds) with symmetrical axial length
enlargement, therefore significant anisometropia is unlikely.
The signs of primary congenital glaucoma include:
an enlarged eye (buphthalmos)
megalocornea
corneal oedema
Haab's striae
corneal scarring and decompensation
immature angle and TM
elevated IOP
cupped optic nerve
myopia and astigmatism
All are true of normal tension glaucoma EXCEPT:
splinter haemorrhages at the disc margin are more common than in POAG
Raynaud's phenomenon and migraine are more common than POAG
nocturnal systemic hypotension is more common than in POAG
the optic nerve head tends to be smaller than in POAG
Your answer was INCORRECT
Explanation
The optic nerve head tends to be larger in NTG than in POAG. All other options are true.
A 58-year-old pseudophakic woman presents with severe pain in her left eye 4 days after a routine
trabeculectomy with mytomycin C. Her visual acuity is hand motions, and IOP 33 mmHg. The
anterior chamber of the left eye is shallow.
The differential diagnosis includes all of the following EXCEPT:
malignant glaucoma
excessive filtration
incomplete iridectomy with obstruction of sclerostomy
delayed suprachoroidal hemorrhage
Your answer was CORRECT
Explanation
Excessive filtration is usually manifested by hypotony and a flat anterior chamber. All other
options provided above can cause a shallow anterior chamber with high pressure.
What is the most distinguishing feature of Cogan-Reese syndrome setting it apart from other ICE
syndromes?
pseudopolycoria
'hammered-silver' corneal endothelium
diffuse iris naevus
corectopia
Your answer was CORRECT
Explanation
The ICE syndrome consists of the following three overlapping disorders:
Progressive iris atrophy: severe iris changes including pseudopolycoria, corectopia, iris atrophy.
Cogan-Reese syndrome: diffuse iris naevus or iris nodules. Iris changes also prominent in 50% of
cases.
Chandler syndrome: hammered-silver corneal endothelial changes. Iris changes present in 40%.
Glaucoma less prominent than the other 2 forms.
All of the following conditions may be associated with blood in Schlemm's canal EXCEPT?
severe thyroid eye disease
carotid-cavernous fistula
excessive digital pressure during gonioscopy with a Goldmann lens
ocular hypertension over 30 mmHg
Your answer was INCORRECT
Explanation
Ocular hypotension (not hypertension) may be associated with blood in Schlemm's canal, which
occurs whenever episcleral venous pressure exceeds intraocular pressure. This may be seen in all
the other conditions mentioned above.
Which gonioscopic method is least likely to distort the anterior chamber anatomy and lead to an
incorrect diagnosis?
Zeiss
Sussman
Koeppe
Goldmann
Your answer was INCORRECT
Explanation
The Koeppe lens is a direct gonioscopy lens (like the Swan-Jacob and the Hoskins-Barkan
models). Direct gonioscopy lenses are used to directly visualise the anterior chamber without the
need for mirrors. They are least likely to distort the anterior chamber anatomy because pressure is
not applied, the lens is typically positioned on the eye with the patient supine while examined by a
portable slit-lamp or an operating microscope. Other options are typically used on the slit lamp.
Posterior pressure on the Goldmann lens which has a large surface area including contact with the
sclera may falsely narrow the angle by indenting the sclera. Lenses such as the Zeiss, Posner and
Sussman models do not utilise a coupling agent (unlike Goldmann); they have a smaller surface
area in contact with the cornea and allow for dynamic assessment of the angle with corneal
indentation. Posterior pressure with these comparatively smaller lenses will push the aqueous from
the center to the periphery of the anterior chamber and may falsely deepen the angle.
Which are the most appropriate initial laser settings for ALT?
Spot size: 50 um, duration 0.5 seconds, energy: 500 mW
Spot size: 500 um, duration 0.1 seconds, energy 200mW
Spot size: 50 um, duration: 0.1 seconds, energy: 700 mW
Spot size: 500 um, duration 0.5 seconds, energy 800 mW
Your answer was CORRECT
Explanation
The small spot size and short duration used in ALT provide great energy in a small area, which is
helpful for a disruptive rather than coagulative effect on the trabecular meshwork.
A similar question came in FRCOphth Part 2, 2016.
Which one of the following statements about ALT is TRUE?
The best location for laser burns with respect to minimizing complications of post-treatment IOP
rise and peripheral anterior synechiae formation is the posterior trabecular meshwork.
Repeating ALT in eyes in which ALT was initially effective and in which IOP control was
eventually lost will provide pressure control in one-third to one-half of cases
The chance of post-treatment IOP rise is not influenced by the number of laser burns applied in
each treatment session
ALT achieves its effect by creating physical openings in the trabecular meshwork through which
aqueous humor can pass from the anterior chamber into Schlemm's canal
Your answer was INCORRECT
Explanation
The technique of creating laser holes through the trabecular meshwork is known as laser
trabeculopuncture (trabeculotomy). This technique was the earliest attempt to treat glaucoma
using laser technology, but it has not been successful in people or in animal models. In ALT, the
risk of a post-treatment IOP rise increases with increasing numbers of laser burns. For this reason,
many clinicians treat 180 degrees of the angle with 50 laser burns in each session. Laser burns
should be placed at the junction of the anterior and posterior pigmented meshwork.
Which topical ocular hypotensive agent should be avoided in children:
pilocarpine
latanoprost
timolol
apraclonidine
Your answer was CORRECT
Explanation
The alpha-2 agonists (apraclonidine and brimonidine) should be avoided in children because they
have the potential to cross the blood-brain barrier.
All are possible precipitating factors for intermittent angle closure EXCEPT:
reading
emotional stress
prone positioning
bright light
Your answer was CORRECT
Explanation
Intermittent episodes of angle closure are precipitated by physiological mydriasis (e.g. dark
conditions) or by physiological shallowing of the anterior chamber when the patient assumes a
prone or semi-prone position (e.g. reading, close-work).
Which statement about selective laser trabeculoplasty (SLT) is FALSE?
SLT disrupts intraocular tissue architecture significantly less than ALT
SLT can be repeated more often than ALT
SLT targets the melanin-containing cells within the trabecular meshwork
SLT has greater IOP-lowering effects compared to ALT
Your answer was CORRECT
Explanation
Selective laser trabeculoplasty (SLT) is a relatively new procedure with IOP-lowering effects that
are comparable to ALT. It has less disruptive effects on intraocular tissue than ALT and can
therefore be repeated more often, up to twice per year.
All are side-effects of carbonic anhydrase inhibitors EXCEPT:
fatigue
hyperglycaemia
tingling of mouth
renal stones
Your answer was CORRECT
Explanation
Carbonic anhydrase inhibitor (CAI) side-effects include:
Paraesthesia of fingers, toes, hands, feet, mucocutaneous junctions
Malaise complex: fatigue, depression, weight loss, reduced libido
GI complex: diarrhea, nausea, cramps
Renal stone formation
Steven-Johnson syndrome
Blood dyscrasias: bone marrow suppression, aplastic anaemia
Topical CAI's may exacerbate corneal decompensation when there is underlying endothelial
dysfunction (rare)
The most common side-effect limiting alpha-2 agonist use is:
bronchoconstriction
bradychardia
allergic conjunctivitis
systemic hypotension
Your answer was CORRECT
Explanation
The major ocular side-effect of the alpha-2 agonists (brimonidine and apraclonidine) is allergic
conjunctivitis, which may be delayed up to a year after starting treatment.
What is the diagnosis in the left eye?
sub-acute angle closure
post-congestive angle closure
primary angle closure glaucoma
acute congestive angle closure
primary angle closure
Your answer was CORRECT
Explanation
This lady has post-congestive angle closure in the right eye. The headache, nausea, Descemet's
folds and glaukomflecken confirm a previous acute angle closure. The pressure is now normal
(hence post-congestive) due to either spontaneous re-opening of the angles (suggested by the fact
that there is irido-corneal touch in only 180 degrees) or due to ciliary body shut-down. She should
be treated with bilateral laser peripheral iridotomies and followed up for signs of glaucoma
progression. In the left eye, she has narrow angles which predispose her anatomically to angle
closure - that is, she has primary angle closure.
Which one of the following is the MOST common cause of glaucoma in eyes with malignant
melanomas?
acute angle closure
tumor cells invading the angle
uveitis
neovascularization
Your answer was CORRECT
Explanation
Malignant melanomas can be associated with normal, elevated, or depressed IOP. Elevated IOP occurs more
frequently with melanomas of the iris/ciliary body than with choroidal melanomas.
Glaucoma may occur by a variety of mechanisms, including:
obstruction of the TM by melanin-containing macrophages (melanomalytic glaucoma)
direct extension of tumor into the trabecular meshwork
angle closure from anterior displacement of the lens-iris diaphragm or PAS
inflammation
neovascularization of the angle.
Neovascularization of the angle is the most common cause of elevated IOP, especially among eyes treated
with radiation
Which is TRUE regarding steroid-induced raised IOP?
there is increased risk with pigmentary glaucoma
it is always reversible with cessation of steroid
it is more common in patients with hypermetropia
it may present within 1 day of starting hourly topical steroid
Your answer was CORRECT
Explanation
Risk factors for steroid-induced raised IOP include:
POAG
pigmentary glaucoma
family history of glaucoma
family history of steroid response
high myopes
diabetics
A pseudophakic patient develops aqueous misdirection syndrome post-trabeculectomy. Treatment
with mydriatic drops and iv mannitol is ineffective.
What is the most appropriate next treatment:
pars plana vitrectomy
Nd:YAG laser of the anterior hyaloid face
bleb needling
peripheral iridoplasty
Your answer was CORRECT
Explanation
Initial treatment of aqueous misdirection syndrome is with topical mydriatics, specifically atropine
1% and phenylephrine 10%. These dilate the ciliary ring and help to direct the ciliary processes
and aqueous anteriorly. IV mannitol can be used if mydriatics are ineffective to reduce the volume
of the vitreous. Subsequent treatment includes Nd:YAG laser of the anterior hyaloid face to break
the ciliary block in pseudophakics. Finally if all else fails, an emergency pars plana vitrectomy
should be performed to excise sufficient vitreous to allow free flow of aqueous anteriorly.
Which is FALSE regarding the treatment of primary congenital glaucoma:
goniotomy is the preferable treatment in a cloudy cornea
trabeculotomy is considered more technically challenging than goniotomy
measurement of corneal diameter is important to monitor response to treatment
trabeculotomy can be combined with trabeculectomy
Your answer was CORRECT
Explanation
Goniotomy is the usual first line of treatment, performed at the time of EUA (examination under
anaesthesia). However, when the cornea is cloudy and the angle difficult to visualize,
trabeculotomy is performed where a scleral flap is raised and a trabeculotome inserted in the canal
of Schlemm. This is a more technically challenging procedure requiring more expertise.
With treatment of congenital glaucoma, optic disc cupping can improve and corneal diameter can
decrease, although increased axial length usually does not change.
A patient who has recently undergone panretinal photocoagulation after a central retinal vein
occlusion presents with neovascularization of the iris and an IOP of 47 mmHg.
All of the following medications are appropriate to use EXCEPT:
apraclonidine
dorzolamide
pilocarpine
atropine
Your answer was CORRECT
Explanation
Pilocarpine is not advocated in the context of intraocular inflammation as it can increase the
chances of posterior synechiae. Moreover, since the trabecular outflow has been occluded by
neovascularization, miotic medications will be ineffective and may, in fact, reduce the uveoscleral
outflow, further elevating the IOP.
In which condition is laser iridotomy LEAST likely to be successful:
intermittent angle closure
Possner-Schlossman syndrome
chronic angle closure
secondary angle closure with pupil block
Your answer was CORRECT
Explanation
Laser iridotomy is indicated in:
angle closure glaucoma - acute, intermittent or chronic
fellow eye of acute angle closure
narrow, occludable angles/ primary angle closure
secondary angle closure with pupil block
POAG with narrow angles or combined mechanism glaucoma
Which of the following is seen in infantile glaucoma but not adult forms of glaucoma?
Haab striae
markedly reduced colour vision
corneal edema
concentric optic nerve cupping
Your answer was CORRECT
Explanation
Figure: Haab striae seen in congenital glaucoma
Haab striae are curvilinear breaks in Descemet's membrane resulting from corneal stretching
induced by high intraocular pressure. They are usually oriented horizontally or concentric to the
limbus in contrast to Descemet's tears resulting from birth trauma, which are typically vertical
Selective laser trabeculoplasty utilises:
green argon laser
blue argon laser
diode laser
Nd:YAG laser
Your answer was CORRECT
Explanation
Selective laser trabeculoplasty utilizes frequency-doubled Q-switched Nd:YAG laser, which
selectively targets melanin pigment in the trabecular meshwork cells. It is potentially safer than
ALT as there is no thermal or structural damage to non-targeted tissue.
For an infant under the age of 1 year, what corneal diameter should be viewed with suspicion for
congenital glaucoma:
greater than 13mm
greater than 11mm
greater than 14mm
greater than 10mm
Your answer was CORRECT
Explanation
Prior to 1 year: >11mm corneal diameter is suspicious
At any age: >13mm corneal diameter is suspicious
At any age: >14mm corneal diameter is typical of advanced buphthalmos
All of the following conditions are typically associated with increased pigmentation of the
trabecular meshwork on gonioscopy EXCEPT:
infantile glaucoma
uveal melanoma
pseudoexfoliation syndrome
prior trauma
pigment dispersion syndrome
Your answer was CORRECT
Explanation
Heavy trabecular meshwork pigmentation is not a feature of anterior segment dysgenesis in infantile
glaucoma.
Heavy pigmentation should suggest the following:
pseudoexfoliation
pigment dispersion
previous inflammation or surgery
uveal melanoma
All are true regarding deep sclerectomy EXCEPT:
the incidence of post-operative hypotony is less than with trabeculectomy
the surgery is technically more challenging than trabeculectomy
longterm results can be enhanced by using a collagen implant
the IOP reduction is greater than with trabeculectomy
Your answer was CORRECT
Explanation
Deep sclerectomy and viscocanalostomy are two techniques of non-penetrating filtration surgery
in that the anterior chamber is not entered at the time of surgery and the internal trabecular
meshwork is preserved. The rate of post-operative hypotony is therefore reduced compared to
conventional trabeculectomy. It is technically more challenging than trabeculectomy. It tends to be
easier to perform in blacks due to the pigmented meshwork improving the visibility of structures.
IOP reduction is generally less than for trabeculectomy, though results can be improved by using a
collagen implant.
Initial treatment of aqueous misdirection syndrome post-trabeculectomy is:
Nd:YAG laser of the posterior hyaloid face
topical mydriatics
topical miotics
pars plana vitrectomy
Your answer was CORRECT
Explanation
Initial treatment of aqueous misdirection syndrome is with topical mydriatics, specifically atropine
1% and phenylephrine 10%. These dilate the ciliary ring and help to direct the ciliary processes
and aqueous anteriorly. IV mannitol can be used if mydriatics are ineffective to reduce the volume
of the vitreous. Subsequent treatment includes Nd:YAG laser of the anterior hyaloid face to break
the ciliary block in pseudophakics. Finally if all else fails, an emergency pars plana vitrectomy
should be performed to excise sufficient vitreous to allow free flow of aqueous anteriorly.
Angle closure secondary to plateau iris is most often caused by what primary underlying anatomic
derangement?
increased relative pupillary block
forward displacement of the ciliary processes
hypertrophic iris pigment epithelium
abnormally thickened peripheral iris stroma
abnormally concave corneoscleral limbus
Your answer was CORRECT
Explanation
Anterior displacement of the ciliary processes pushes the peripheral iris forward, which
subsequently narrows the anterior chamber and blocks the trabecular meshwork. This has been
confirmed with anterior segment OCT. Patients who have plateau iris syndrome are often resistant
to laser peripheral iridotomy and should be treated with long-term miotics and possibly iridoplasty
to mechanically shrink the peripheral iris away from the meshwork.
Structures visible on gonioscopy from anterior to posterior are:
Schwalbe's line, pigmented trabecular meshwork, non-pigmented trabecular meshwork, scleral
spur, ciliary body
Schwalbe's line, non-pigmented trabecular meshwork, pigmented trabecular meshwork, ciliary
body, scleral spur,
Schwalbe's line, non-pigmented trabecular meshwork, pigmented trabecular meshwork,
scleral spur, ciliary body
Schwalbe's, pigmented trabecular meshwork, non-pigmented trabecular meshwork, ciliary body,
scleral spur
Your answer was CORRECT
Explanation
The order of visible structures on gonioscopy from anterior to posterior are: Schwalbe's line, non-
pigmented trabecular meshwork, pigmented trabecular meshwork, scleral spur, ciliary body
What is the mechanism of IOP-lowering by pilocarpine in open-angle glaucoma?
it increases aqueous outflow via the uveo-scleral route
it increases aqueous outflow via the trabecular meshwork
it reduces aqueous secretion
it physiologically deepens the irido-corneal drainage angle
Your answer was CORRECT
Explanation
Miotic agents lower intra-ocular in primary open-angle glaucoma by their action on the ciliary
muscle, which upon contraction opens the trabecular meshwork and increases aqueous outflow. In
angle-closure glaucoma miotic agents are effective by pulling the peripheral iris away from the
angle, thus opening the angle.
A 54-year-old man presents with IOPs of 24 mm Hg in the right eye and 26 mm Hg in the left eye.
His visual fields and optic nerve heads appear normal.
All of the following are recognised risk factors for glaucoma that might prompt institution of
prophylactic medical therapy EXCEPT?
elevated nerve fiber layer on OCT
high myopia
family history of glaucoma
reduced central corneal thickness pachymetry measurements
African American descent
Your answer was CORRECT
Explanation
Treatment of the suspected patient with glaucoma should be limited to patients with a high risk of ultimate
damage to the optic nerve. These risk factors include:
elevated IOP
positive family history of glaucoma
myopia
diabetes mellitus
cardiovascular disease
African American race
asymmetric cupping
large cups
early nonspecific visual field changes
thin corneas
Decreased nerve fiber layer (NFL) thickness readings using OCT would make one suspicious for optic nerve
damage.
Which statement is FALSE regarding argon laser trabeculoplasty?
it is more effective in Afro-Caribbean patients than Caucasians
the laser burns are placed at the junction of the pigmented and non-pigmented trabecular
meshwork
the failure rate is 30% at 5 years
it is more effective in pseudoexfoliation glaucoma compared to POAG
Your answer was CORRECT
Explanation
The failure rate is about 50-65% at 5 years. Other options above are true.
Glaucoma which manifests at 24 months of age is classified as:
infantile glaucoma
late-onset congenital glaucoma
juvenile glaucoma
true congenital glaucoma
Your answer was CORRECT
Explanation
Congenital glaucoma is classified as:
True congenital glaucoma (40%) which develops in utero
Infantile glaucoma (55%) which develops before the third birthday
Juvenile glaucoma (5%) develops after the third birthday but before the age of 16 years
All are true of pseudoexfoliation EXCEPT:
it is particularly common in Scandinavia
it is more common in males
it is a systemic disorder
it is produced by abnormal basement membranes of ageing epithelial cells
Your answer was CORRECT
Explanation
Pseudoexfoliation material is produced by abnormal basement membranes of ageing epithelial
cells in the trabeculum, lens capsule, iris and ciliary body. It is more common in females. Other
options above are true.
Which statement about anti-metabolite glaucoma therapy is FALSE?
MMC directly interacts with DNA, blocking RNA and subsequent protein synthesis in all
metabolically active cells
great care should be taken to prevent exposure of the bleb wound edges to MMC in order to
prevent bleb leak
5-FU is associated with a lower incidence of postoperative hypotony and surface-related
complications than MMC
intra-operative dosage of MMC varies from 0.2 to 0.5 mg/ml for 1 to 4 minutes of exposure,
depending on the specific clinical situation
the anti-proliferative effects of MMC are generally more potent and longer-lasting than those of a
comparable dose of 5-FU
Your answer was CORRECT
Explanation
MMC is associated with a lower incidence of hypotony and corneal-surface disorders than 5-
fluorouracil. Other options supplied are true.
The most effective treatment to prevent the progression of rubeosis iridis to neovascular glaucoma
is:
pan-retinal photocoagulation
iridoplasty
anti-VEGF
pars plana vitrectomy
Your answer was CORRECT
Explanation
Pan-retinal photocoagulation is the most important early, definitive treatment for rubeosis iridis. If
performed early, it can induce regression of vessels and prevent the development of neovascular
glaucoma. Other treatment options to consider for rubeosis include anti-VEGF injections, and
depending on the aetiology (e.g. post-diabetic, post-retinal detachment) pars plana vitrectomy.
What is the most common late complication of glaucoma drainage implants?
bleb encapsulation
malposition
blockage of the tube end with blood, vitreous or iris
tube erosion
excessive drainage
Your answer was CORRECT
Explanation
All options above are possible complications of glaucoma tube surgery. However, the most
common late complication, occurring in 10% of cases is bleb encapsulation.
What is the rate of blebitis in trabeculectomy with antimetabolite augmentation?
5%
1%
2%
0.1%
Your answer was CORRECT
Explanation
The rate of blebitis following trabeculectomy with anti-metabolite augmentation is approximately
5%. The rate of endophthalmitis is less than 1%.
All of the following features on clinical examination should raise doubts about the diagnosis of
glaucomatous optic nerve damage EXCEPT?
markedly asymmetric colour visual loss
field defects obeying the vertical meridian
presence of afferent pupillary defect
pallor out of proportion to cupping
Your answer was CORRECT
Explanation
The following features should raise suspicion about the diagnosis of glaucoma:
an optic disc that is less cupped than would be expected for observed field loss
pallor of the disc that is more impressive than the cupping
markedly asymmetric dyschromatopsia
visual field defects uncharacteristic for glaucoma (e.g., respecting the vertical meridian)
A relative afferent pupillary defect (RAPD) can occur and in fact would be entirely consistent with a
diagnosis of asymmetric glaucoma.
A patient has had a complicated retinal detachment repaired with silicone oil instillation. Which
one of the following is TRUE concerning the peripheral iridectomy?
it prevents pupillary block glaucoma that can occur with silicone oil
it allows for a much-needed alternate pathway for light entry into the eye
it prevents neovascularization of the iris
the iridectomy should ideally be performed superiorly
Your answer was CORRECT
Explanation
A large inferior peripheral iridectomy is advised when silicone oil is placed into an eye,
particularly an aphakic eye that is at risk of pupil block. The oil is lighter than water, and to
prevent pupillary block glaucoma, an inferior peripheral iridectomy is performed.
All are true of iridocorneal endothelial (ICE) syndrome EXCEPT:
corectopia occurs in essential iris atrophy but not Cogan-Reese syndrome
in Chandler's syndrome, corneal oedema may occur with normal IOP
it is almost always unilateral
it affects women more frequently than men
Your answer was CORRECT
Explanation
While iris atrophy and changes such as corectpia are most evident in essential iris atrophy, there is overlap in
the clinical features of all 3 sub-types of ICE syndrome such that iris changes can be seen in all three sub-
types.
The clinical variants of ICE syndrome are:
Progressive iris atrophy: severe iris changes including atrophy, corectopia and pseudopolycoria
Cogan-Reese syndrome: diffuse iris naevus covering the anterior iris or iris nodules with or without
iris atrophy
Chandler syndrome: hammered-silver corneal endothelial abnormalities with corneal
decompensation
Relative to patients with primary open-angle glaucoma, the following conditions are all associated
with a poorer response to argon laser trabeculoplasty EXCEPT?
pseudoexfoliative glaucoma
uveitic glaucoma
angle recession glaucoma
aphakic glaucoma
Your answer was CORRECT
Explanation
ALT response is better for pigmentary glaucoma and pseudoexfoliation compared to POAG.
ALT response is poorer for:
inflammatory diseases
recessed angles
membranes in angles
young patients with developmental defects
aphakic eyes
A 54-year-old woman with insulin-dependent diabetes mellitus has a non-clearing vitreous
hemorrhage in her left eye and proceeds to vitrectomy. One week after surgery, she presents with
left eye pain, an IOP of 58 mmHg, and a candy-stripe hyphaema.
The most likely diagnosis is:
phacolytic glaucoma
ghost cell glaucoma
haemolytic glaucoma
phacomorphic glaucoma
Your answer was CORRECT
Explanation
The candy stripe is a tip-off to ghost cell glaucoma. Two conditions are necessary for the
development of this unique form of glaucoma: vitreous haemorrhage (red blood cells degenerate
and become rigid in the vitreous) and a break in the anterior hyaloid face (to allow the cells to
enter the anterior chamber). The khaki-coloured ghost cells layer out in the anterior chamber and
can be distinguished from fresher red cells, creating the effect of a candy stripe. Ghost cells cannot
escape easily from the meshwork and hence produce trabecular obstruction and raise the IOP.
In haemolytic glaucoma, haemoglobin-laden macrophages block the meshwork, while in
phacolytic glaucoma, macrophages are engorged with lens protein from a hypermature cataract.
A 54-year-old woman with insulin-dependent diabetes mellitus has a non-clearing vitreous
hemorrhage in her left eye and proceeds to vitrectomy. One week after surgery, she presents with
left eye pain, an IOP of 58 mmHg, and a candy-stripe hyphaema.
The most likely diagnosis is:
phacolytic glaucoma
ghost cell glaucoma
haemolytic glaucoma
phacomorphic glaucoma
Your answer was CORRECT
Explanation
The candy stripe is a tip-off to ghost cell glaucoma. Two conditions are necessary for the
development of this unique form of glaucoma: vitreous haemorrhage (red blood cells degenerate
and become rigid in the vitreous) and a break in the anterior hyaloid face (to allow the cells to
enter the anterior chamber). The khaki-coloured ghost cells layer out in the anterior chamber and
can be distinguished from fresher red cells, creating the effect of a candy stripe. Ghost cells cannot
escape easily from the meshwork and hence produce trabecular obstruction and raise the IOP.
In haemolytic glaucoma, haemoglobin-laden macrophages block the meshwork, while in
phacolytic glaucoma, macrophages are engorged with lens protein from a hypermature cataract.
Which side-effect of glaucoma medication is correctly matched?
timolol and palpitations
dipivefrin and cystoid macular oedema
bimatoprost and reduced periorbital skin pigmentation
pilocarpine and reduced vascular permeability
Your answer was CORRECT
Explanation
Dipivefrin (epinephrine), a somewhat outdated glaucoma medication, is known to cause
conjunctival injection and cystoid macular oedema, particularly in aphakic patients.
Bimatoprost is associated with increased pigmentation, hypertrichosis and periorbital fat atrophy.
Timolol may cause bradycardia and depression (not palpitations).
Pilocarpine may cause increased vascular permeability and should therefore be avoided in the
context of uveitis.
This question came in the FRCS (Glasgow) Part 2 in October 2014.
A 70-year-old man originally from Bangladesh presents with a painful, red eye. He has had poor
vision in this eye for years, but he feels it has worsened over the past 2 weeks. He denies any
ocular trauma. On examination, he is noted to have epithelial oedema, heavy flare and cells, and a
white cataract. The IOP is 56 mmHg.
What is the most likely diagnosis?
phacoanaphylactic glaucoma
phacolytic glaucoma
uveitic glaucoma
phacomorphic glaucoma
Your answer was CORRECT
Explanation
Phacolytic glaucoma occurs when lens proteins leak through an intact lens capsule, inducing a
heavy macrophage response. The trabecular meshwork is obstructed with high molecular weight
lens proteins and bloated macrophages. It usually occurs in older patients with a mature,
hypermature, or even Morgagnian cataract. The lens needs to be removed to cure this problem.
Phacomorphic glaucoma occurs in patients with large crystalline lenses, causing pupillary block
and secondary angle closure.
Phacoanaphylactic glaucoma occurs after a traumatic injury ruptures the lens capsule, allowing a
granulomatous inflammation and secondary glaucoma.
Lens-particle glaucoma occurs when remnants of lens material remain after cataract surgery,
causing inflammation and raised pressure.
A man with glaucoma has a paracentral scotoma and a visual field with an MD of -4 dB.
What grade of glaucomatous damage does he have?
Grade 3
Grade 1
Grade 2
Grade 4
Your answer was CORRECT
Explanation
Grading of glaucomatous damage:
Mild (Grade 1): minimal cupping, a nasal step or paracentral scotoma and a MD < -6dB
Moderate (Grade 2): thinning of neuroretinal rim, an arcuate scotoma and a MD < -12dB
Severe (Grade 3): marked cupping, extensive VF loss including the central 5 degrees and MD > -
12dB
End-Stage (Grade 4): gross disc cupping and a small residual field
Which medication can cause acute angle-closure glaucoma?
vigabatrin
sodium valproate
topiramate
chlorpromazine
Your answer was CORRECT
Explanation
Topiramate is an anti-epileptic which can cause acute angle-closure glaucoma by causing swelling
and rotation of the ciliary body. The reaction is idiosyncratic, not dose-related, and can occur
within 2 weeks of commencing the medication. Other ocular side-effects of topiramate include
acute myopia, scleritis, blepharospasm, oculogyric crises and peri-orbital oedema.
Which of the following techniques is LEAST helpful in evaluating the appearance of a
glaucomatous optic nerve:
slit-lamp examination using a 90-D lens
indirect ophthalmoscopy using a 20-D lens
slit-lamp examination using a contact lens
direct ophthalmoscopy
Your answer was CORRECT
Explanation
The small image obtained using indirect ophthalmoscopy does not allow for adequate evaluation
of the optic nerve details.
Careful examination with the direct ophthalmoscope can provide important information about the
pallor of the optic cup; however, the most effective methods include stereoscopic examination
using the slit lamp in combination with a posterior-pole contact lens, a 90-D lens, or a Hruby lens.